You are on page 1of 117

Government Property

NOT FOR SALE

NOT
10
Mathematics 11
Quarter 3 - Module 1
Solving Problems Involving
Permutations

Department of Education ● Republic of the Philippines


Mathematics- Grade 10
Alternative Delivery Mode
Quarter 3- Module 1: Solving Problems Involving Permutations
First Edition, 2020

Republic Act 8293, section 176 states that: No copyright shall


subsist in any work of the Government of the Philippines. However, prior
approval of the government agency or office wherein the work is created shall
be necessary for exploitation of such work for profit. Such agency or office
may, among other things, impose as a condition the payment of royalty.

Borrowed materials (i.e., pictures, photos, brand names, trademarks,


etc.) included in this book are owned by their respective copyright holders.
Every effort has been exerted to locate and seek permission to use these
materials from their respective copyright owners. The publisher and authors
do not represent nor claim ownership over them.

Published by the Department of Education

Secretary: Leonor Magtolis Briones


Undersecretary: Diosdado M. San Antonio

Development Team of the Module


Author/s: Jezza C. Biaῆo, T1
Reviewers: Rhodel A. Lamban, PhD
Elbert R. Francisco, PhD
Renie Nicolas B. Ontanillas
Vina Marie B. Latras
Illustrator and Layout Artist: Sheen Kenlord F. Aquino
Management Team

Chairperson: Arturo B. Bayocot, PhD CESO III


Regional Director

Co-Chairpersons: Victor G. De Gracia Jr., PhD CESO V


Asst. Regional Director

Randolph B. Tortola, PhD, CESO IV


Schools Division Superintendent

Shambaeh A. Usman, PhD


Assistant Schools Division Superintendent
Mala Epra B. Magnaong, Chief ES, CLMD
Neil A. Improgo, PhD EPS-LRMS
Bienvenido U. Tagolimot, Jr., PhD EPS-
ADM Members: Elbert R. Francisco, PhD, Chief Dach, CID
Rhodel A. Lamban, PhD, EPS Mathematics
Rejynne Mary L. Ruiz, PhD, LRMDS
Manager
Jeny B. Timbal, PDO II
Shella O. Bolasco, Division Librarian II

Printed in the Philippines by


Department of Education – Division of Bukidnon
Office Address: Fortich Street, Sumpong, Malaybalay City
Telephone: (088) 813-3634
E-mail Address: bukidnon@deped.gov.ph
10
Mathematics
Quarter 3 - Module 1
Solving Problems Involving
Permutations

This instructional material was collaboratively developed and


reviewed by educators from public schools. We encourage teachers
and other education stakeholders to email their feedback, comments,
and recommendations to the Department of Education at
bukidnon@deped.gov.ph.

We value your feedback and recommendations.

Department of Education-Division of Bukidnon ● Republic of the


Philippines
Table of Contents

What This Module is About


Note to the Teacher/Facilitator
Note to the Learner
Note to the Parents/Guardian
What I Need to Know
How to Learn from this Module
Icons of this Module

What I Know (Pre-test)

Lesson 1: Page
Solving Problems in Permutations using the
Fundamental Counting Principle – Day 1 1

What I Need To Know 1

What I Know 1

What’s In 3

What’s New 4

What Is It 7

What’s More 8
Guided/Controlled Assessment
Independent Practice

What I Have Learned 15

What I Can Do 15

Assessment 17
Guided Assessment
Independent Assessment

Additional Activities 24

Lesson 2:
Solving Problems in Permutations using the
Permutations Formula – Day 2 25

What I Need To Know 25

What I Know 25

What’s In 27

What’s New 28
Page

What Is It 30

What’s More 32
Guided/Controlled Assessment
Independent Practice

What I Have Learned 38

What I Can Do 39

Assessment 40
Guided Assessment
Independent Assessment

Additional Activities 46

Lesson 3:
Distinguishable and Circular Permutations – Day 3 47

What I Need to Know 47

What I Know 47

What’s In 49

What’s New 50

What Is It 54

What’s More 57
Guided/Controlled Practice
Independent Practice

What I Have Learned 63

What I Can Do 64

Assessment 65
Guided Assessment
Independent Assessment

Additional Activities 71

Summary 72

Assessment (Post-Test) 73

Answer Keys 75

References 79
What This Module is About
This module is about solving problems involving permutations.
This will require the knowledge and skill of the basic counting technique
called the Fundamental Counting Principle. As you go over the
discussion and exercises, you will understand and know how to solve
different problems using the permutations formula and distinct or
distinguishable permutations and circular permutations. With the
dissimilar activities or exercise being furnished of this module, may you
find this material challenging and engaging as it develops your critical-
thinking and problem-solving skills. Enjoy reading with learning and
solving every exercises and do not doubt to go back if you miss some
items.

We hope that through this module, you will experience meaningful


learning and acquire deep understanding of this significant
competency.

For the Teacher/Facilitator:


Welcome to the Mathematics 10 Alternative Delivery Mode (ADM)
Module on Solving Problems Involving Permutations!
This module was collaboratively designed, developed and reviewed by
educators both from public and private institutions to assist you, the
teacher or facilitator in helping the learners meet the standards set by
the K to 12 Curriculum while overcoming their personal, social, and
economic constraints in schooling.

As a teacher or a facilitator, you are expected to help and orient


the learners on how to use this module. You also need to keep track of
the learners' progress while allowing them to manage their own
learning. Furthermore, you are expected to encourage and assist the
learners as they do the tasks included in the module. In addition,
teachers should put hard work in taking good care of our learners
because they are the hopes of our nation.

For the Learner:


Welcome to the Mathematics 10 Alternative Delivery Mode (ADM)
Module on Solving Problems Involving Permutations!
This module are for you to provide fun and meaningful opportunities for
guided and independent learning at your own speed and time. You will
be enabled to process the contents of the learning resource while being
an active learner.

As a learner, you are expected to master this competency by


making an effort to read, write and understand as long as solve
problems in real-life situations with all your sincerity and honesty
cooperation. Use your mind and hand in working this module because
through your mind and hands you may learn, formulate, perform and
achieve. The mind and hand is important, but the most important of
them is our heart that has willingness and love to work with this
material. Thus, your academic success lies in you!

For the Parents/Guardians:


Hello my dear parents/guardians, Welcome to the Mathematics
10 Alternative Delivery Mode (ADM) Module on Solving Problems
Involving Permutations!

This module was designed for your sons and daughters for them
to learn and finish schooling even though they are at home.
As a parent/guardian, I am asking your guidance and help to work out
the lessons and exercises together. I know you are busy working for
financial support but give some time to know your child if they read this
module and answer the different exercises or activities. If your child
approach you to help some of the problems that they don’t understand,
try to reach them out and have your full support to finish this material.
Let us help and work together for the future of success of our dear
students because I believe that by working together, we can make your
child’s experience positive and successful. And don’t forget, gently
encourage your child to do their best.

What I Need to Know


This module was designed and written with you in mind. It is here
to help you master in solving problems involving permutations. The
range of this module permits it to be used in many different problems
solving. The language used recognizes the different vocabulary level of
students. The lessons are arranged to follow the standard sequence of
the course.

The module has three lessons, namely:


 Lesson 1 – Solving Problems in Permutations using the Fundamental
Counting Principle
 Lesson 2 – Solving Problems using the Permutations Formula
 Lesson 3 – Solving Problems in Distinguishable and Circular
Permutations

After going through this module, the learner shall be able to:

1. Define the Fundamental Counting Principle;


2. Solve problems in permutations using the fundamental counting
principle;
3. Solve problems using permutations formula;
4. Define the distinct or distinguishable and circular permutations; and
5. Solve problems using distinct or distinguishable and circular
permutations formula.

How to Learn from this Module


To achieve the objectives cited above, you are to do the following:

 Use the module with care.


 Do not put unnecessary mark/s on any part of the module.
Use a separate sheet of paper in answering the activities.

• Take your time reading the lessons carefully and understand every
lessons.
• Follow the directions and/or instructions in the activities and exercises
diligently.
 Observe honesty and integrity in doing the tasks and checking your
answers.

• Answer all the given tests and exercises. Do not leave the problems
unanswered.
• Return this module to your teacher/facilitator once you are through with
it.

If you have any difficulty in answering the exercises in this


module, do not hesitate to approach your teacher or facilitator. Always
put in mind that you are not alone. I know you can do it!
Icons of this Module
What I
Need to This part contains learning objectives that
Know are set for you to learn as you go along the
Module each day/lesson.

What I
know This is a pre-test assessment as to your level of
knowledge to the subject matter at hand,
meant specifically to gauge prior related
Knowledge
What’s
In This part connects previous lesson with that
of the current one.

What’s
New An introduction of the new lesson through
various activities, before it will be presented
to you.

What is
It These are discussions of the activities as a
way to deepen your discovery and under-
standing of the concept.

What’s
More These are follow-up activities that are in-
tended for you to practice further in order to
master the competencies.

What I
Have Activities designed to process what you
Learned have learned from the lesson

What I
can do These are tasks that are designed to show-
case your skills and knowledge gained, and
applied into real-life concerns and situations.
What I Know

PRE-ASSESSMENT

Find out how much you already know about this module. Choose
the letter that you think is correct and write your answer on the space
provided before each number on the separate sheet. Take note of the
items that you were not able to answer correctly and look for the right
answer as you go through this module.

___1. Find the number of distinguishable permutations of the letters of


the word PASS.
A. 4 B. 12 C. 36 D. 144
___2. In how many ways can 8 people be seated around a circular
table if two of them insist on sitting beside each other?
A. 360 B. 720 C. 1440 D. 5040
___3. How many different 4-digit even numbers can be formed from the
digits 1, 3, 5, 6, 8, and 9 if no repetition of digits is allowed?
A. 1 680 B. 840 C. 420 D. 120
___4. Ms. Dela Cruz wants to produce different sets of test questions
for her essay test. If she plans to do this by putting together 3 out
of 5 questions she prepared, how many different sets of
questions could she construct?
A. 10 B. 20 C. 60 D. 80
___5. In a town fiesta dancing competition with 12 contestants, in how
many ways can the organizer arrange the first three dancers?
A. 132 B. 990 C. 1320 D. 1716
___6. Which of the following expressions represents the number of
distinguishable permutations of the letters of the word
HAPPINESS?
9! 9! 9!
A. 9! B. C. D.
2! 5! 2 ! 2! 2 !
___7. How many different 3-digit numbers can be formed from the
digits 1,2,3,5,6,7 if repetition of digits is not allowed?
A. 120 B. 360 C.720 D.840
___8. Determine the different ways can 6 potted plants be arranged in
a
row?
A. 210 B. 540 C. 550 D. 720
___9. In how many different ways can 5 different-colored horses be
positioned in a carousel?
A. 24 B. 120 C. 720 D. 800
___10. In a room, there are 10 chairs in a row. In how many ways can
5
students be seated in consecutive chairs?
A. 120 B. 720 C. 25 600 D. 30 240
___11. In how many ways can 4 people be seated around a circular
table?
A. 2 B. 6 C. 12 D. 24
___12. Lydia, together with her five friends want to arrange themselves
around a round table. How many ways can they do it?
A. 720 B. 120 C. 60 D. 30
___13. In the recently conducted SEA Games, there are 11 contenders
in a swimming events. How many ways can the gold, the silver,
and the bronze medals be won?
A. 856 B. 878 C. 950 D. 990
___14. Six athletes will participate in a gymnastics competition. In how
many ways can they win first, second, and third places, if there
are no ties?
A. 110 B. 120 C. 210 D. 150
___15. Find the number of distinguishable permutations of the letters of
the word EDUCATED.
A. 1680 B. 10 080 C. 20 160 D. 40 320
Solving Problems in
Permutations using
the Fundamental
Lesson Counting Principle
1
What I Need to Know
This lesson is written for you to solve problems on permutations
using the basic counting technique called the Fundamental Counting
Principle. We can either use reasoning to solve these types of
permutation problems. Basically, in fundamental counting principle is
you multiply the events together to get the total number of outcomes .

You may SKIP this lesson or its activities if you already master the skill
and get a score of 11 out of 15 items in the pre-test.

What I Know
Find out how much you already know about this lesson. Choose
the letter that you think is correct and write your answer on the space
provided before each number on the separate sheet. Take note of the
items that you were not able to answer correctly and look for the right
answer as you go through this module.

___1. If Gerald has 10 T-shirts, 7 pairs of pants, and 4 pairs of shoes,


how many possibilities can he dress himself up for the day?
A. 280 B. 290 C. 380 D. 390

___2. Twelve students compete in a race. In how many ways first three prizes
be given?
A. 1 000 B. 1 220 C. 1 320 D. 1 420

___3. You have 10 pairs of pants, 6 shirts, and 3 jackets. How many
outfits can you have consisting of a shirt, a pair of pants, and a
jacket?
A. 60 B. 120 C. 180 D. 240

1
___4. How many four-digit numbers can be formed from the numbers
1, 3, 4, 6, 8, and 9 if repetition of digits is not allowed?
A. 350 B. 360 C. 370 D. 380

___5. From among the 36 teachers in a college, one principal, one


vice-principal and the teacher-in charge are to be appointed. In
how many ways this can be done?
A. 41 800 B. 41 840 C. 42 800 D. 42 840

___6. In how many ways can 4 employees be chosen (from 12) to be


placed in 4 different jobs?
A. 11 880 B. 11 900 C. 12 880 D. 12 900

___7. How many different ways can the offices of president, vice
president, secretary, and treasurer be chosen from an
organization of 67 members?
A. 17 395 520 B. 18 395 520
C. 19 395 520 D. 20 395 520

___8. In how many ways can 5 motorcycles be parked if there are 7


available parking spaces?
A. 1 250 B. 1 520 C. 2 250 D. 2 520

___9. How many different ways are there to arrange your first three
classes if they are math, science, and language arts?
A. 2 B. 4 C. 6 D. 8

___10. You take a survey with five “yes” or “no” answers. How many
different ways could you complete the survey?
A. 12 B. 24 C. 28 D. 32

___11. It is required to seat 5 men and 4 women in a row so that the


women occupy the even places. How many such arrangements
are possible?
A. 2 880 B. 2 900 C. 3 880 D. 3 900

___12. Using the digits 1, 2, 3, and 5, how many 4 digit numbers can be
formed if the number must be divisible by 2 and repetition is not
allowed?
A. 6 B. 12 C. 24 D. 36

___13. There are four Gators in a holding cell at the jail. They will be
asked to arrange themselves from left to right in a police line-up.
How many different line-ups are possible?
A. 6 B. 12 C. 24 D. 36
2
___14. 10 students have appeared in a test in which the top three will
get a prize. How many possible ways are there to get the prize
winners?
A. 620 B. 720 C. 820 D. 920

___15. There are 6 people who will sit in a row but out of them Ronnie
will always be left of Annie and Rachel will always be right of
Annie. In how many such arrangement can be done?
A. 120 B. 160 C. 200 D. 220

I hope you answered all of the item! To check your score!


If your score is:

11- You are doing great! You may proceed to the next
15 activity that can be found in What’s More after reviewing
the missed items.
6-10 You are doing well but you need to proceed to the next
activities/exercises of this lesson.
1-5 You need to study harder. Ask help from your teacher or
parent or friend. Study the next examples and answer the
next activities.

What’s In

In this lesson, it is relevant to know the basic operation in


Mathematics especially the multiplication and addition. I think you
already mastered it when you were in your elementary level. The
lesson is not just multiplies and adding all the numbers but also
comprehending first the problems before performing the operations.
3
What’s New
Activity 1: Find Me Interesting!

1. In how many ways can four-letter word CODE be arrange its letters 3
at a time?

Solution: Using Reasoning

There are 3 positions to be filled. We write the horizontal marks:


__ __ __. On each horizontal mark, we write the number of choices we
have in filling up the said position. There are 4 choices for the first
position and 3 choices left for the second position. There are 2 choices
left for the third position after filling the first and second position.
Therefore, the three entries on the horizontal marks would be

4 3 2

We apply the Fundamental Counting Principle and multiply:

4 ×3 ×2=24

Thus, there are 24 possible ways of arranging the 4 letters of


CODE taking 3 at a time. We also say the 4 letters of CODE, taken 3 at
a time, have 24 permutations.

2. Twelve students compete in a race. In how many ways first


three prizes be given?

Solution: Using Reasoning

The total number of students is 12. All the students will have
equal chance to get 1st prize.

The number of ways to get the first prize is 12.

Out of 12 students, only 11 students are eligible to get the second


prize. Because 1 student got the first prize already. 

The number of ways to get the second prize is 11.

Out of 12 students, only 10 students are eligible to get the second


prize. Because for the 2 students already got the first and second prize
respectively.

4
Number of ways to get the third prize = 10

Total number of ways = 12 x 11 x 10 = 1320

There are 1320 number of ways.

3. In how many ways can a president, a treasurer and a secretary be


chosen from among 7 candidates?

Solution: Using Reasoning

For the first position, there are 7 possible choices. After that
candidate is chosen, there are 6 possible choices. Finally, there are 5
possible choices. We apply the Fundamental Counting Principle and
multiply:

7 × 6 × 5 = 210

There are 210 possible ways to choose a president, a treasurer


and a secretary be chosen from among 7 candidates.

To test if you understand the examples above, answer the activity


below. You can do this!

Activity 2: Find Me Out!

Answer each permutations completely: Show your solutions to the


other sheets given.

1. A license plate begins with three letters. If the possible letters are A, B, C,
D and E, how many different permutations of these letters can be made if
no letter is used more than once?

2. In how many ways can 6 people arrange themselves in a row for picture
taking?

3. In a town fiesta singing competition with 10 contestants, in how many


ways can the organizer arrange the first three singers?

4. Determine the different ways can 7 potted plants be arranged in a


row?

5
5. Using the digits 1, 2, 3, and 5, how many 4 digit numbers can be
formed if the number must be divisible by 2 and repetition is not
allowed?

6. A doctor wants to assign 4 different tasks to his 4 nurses. In how


many possible ways can he do it?

7. In how many ways can 5 motorcycles be parked if there are 7


available parking spaces?

8. In how many different ways can 13 people occupy the 13 seats in a


front row of an Amphitheater?

9. In how many ways can you place 10 different books on a shelf if


there is space enough for only 6 books?

10. Ten swimmers join a swimming competition. In how many ways can
they be arranged as first, second, and third placers?

6
What Is It
Fundamental Counting Principle Defined

Fundamental counting principle states that activity A can be done


in n1 ways, activity B can be done in n2 ways, activity C in n3 ways, and
so on, then activities A, B, C can be done simultaneously in
n1 ×n 2 × n3 … ways.

For example:
A. A close friend invited Marie on her birthday party. Marie has 4 new
dresses (sleeveless, stripes, long-sleeved, and with ruffles) and 3
shoes (black, pink, and blue) in her closet reserved for such
occasions.

1. In how many ways can Marie select her outfit if any shoes can be
paired with any dress? List all the possibilities.

Solution:

Dresses- sleeveless, stripes, long-sleeved, with ruffles


Shoes- black, pink, blue
Possible outfits: Possible outfits:
Dress - shoe Dress - shoe
Sleeveless - black long-sleeved- black
Sleeveless - pink long-sleeved- pink
Sleeveless - blue long-sleeved- blue
Stripes - black with ruffles - black
Stripes - pink with ruffles - pink
Stripes - blue with ruffles - blue

2. How many dress and shoe pairs are possible?


12 dress and shoe pairs are possible.

In this example, you can also use the tree diagram in finding the
possible pairs of dress and shoes. But the number of possible dress-
shoes pairs can be obtained also by using the fundamental counting
principle (FCP).

Solution: 4 choices for dress x 3 choices of shoes = 12 possible


pairs
7
B. A person wants to buy one fountain pen, one ball pen and one
pencil from a stationery shop. If there are 10 fountain pen varieties,
12
ball pen varieties and 5 pencil varieties, in how many ways can he
select these articles?

Solution:

A person need to buy fountain pen, one ball pen and one pencil.
That is we have to do all the works.

Number of ways selecting fountain pen = 10

Number of ways selecting ball pen = 12

Number of ways selecting pencil = 5

Total number of selecting all these = 10 x 12 x 5 = 600 number of


ways

Therefore, there are 600 number of ways of selecting a fountain


pen, ball pen, and a pencil.

What’s More
Activity 3: Guided/Controlled Practice

Study the different problems with solution below. If there are


some item that you don’t understand, do not hesitate to ask assistance
from your teacher’s, parent, and friend.

1. How many ways can I arrange the letters A, B, C, D, E, and F?

Solution:
6 ×5 × 4 × 3× 2× 1=720

2. You are choosing a password that has 4 letters followed by 2


digits. How many passwords are possible if letters and digits can
be repeated?

Solution:

There are 26 choices for each letter and 10 choices for each digit.
Use the fundamental counting principle.
8
Number of passwords = 26 ×26 × 26 ×26 ×10 × 10=45697 600

Thus, the number of different passwords is 45 697 600.

3. To buy a computer system, a customer can choose one of 4


monitors, one of 2 keyboards, one of 4 computers and one of 3
printers. Determine the number of possible systems that a
customer can choose from.

Solution:

Using the counting principle used in the introduction above, the


number of all possible computer systems that can be bought is given
by:

N=4 × 2× 4 × 3=96

Therefore, the number of possible systems that a customer can


be choose from is 96.

4. A student can select one of 6 different mathematics books, one


of three different chemistry books and one of 4 different science
books. In how many different ways can a student select a book of
mathematics, a book of chemistry and a book of science?

Solution:

The total number N of different ways that the students can select
his 3 books is given by:

N=6 ×3 × 4=72

So, the number N of different ways that the students can select
his 3 books is 72.

5. There are 3 different roads from city A to city B and 2 different


roads from city B to city C. In how many ways can someone go
from city A to city C passing by city B?

Solution:

The total number N of different ways that someone can go from


city A to city C, passing by city B is:

N=3× 2=6

Thus, the total number N of different ways that someone can go


from city A to city C, passing by city B is 6.
9
6. A man has 3 different suits, 4 different shirts and 5 different pairs
of shoes. In how many ways can this man wear a suit, a shirt and
a pair of shoes?

Solution:

The total number N of different ways that this man can wear one
of his suits, one of his shirts and a pair of his shoes is:

N=3× 4 × 5=60

Therefore, the total number of different ways that this man can
wear one of his suits, one of his shirts and a pair of his shoes is 60.

7. In a company, ID cards have 5 digits numbers. How many ID


cards can be formed if repetition of the digit is allowed?

Solution:

Any of the five digits of the number to be formed can be any of the
10 digits: 0, 1, 2, 3, 4, 5, 6, 7, 8, 9. Hence the 10 choices for each digit
of the number to be formed since repetition of the digits from 0 to 9 is
allowed. When repetition is allowed, the total number N of ID cards is
given by the total numbers of 5 digit numbers that can formed and is
given by:

N=10 × 10× 10 ×10 ×10=100000

The total numbers of 5 digit numbers that can be formed is 100


000.

8. Using the digits 1, 2, 3, and 5, how many 4 digit numbers can be


formed if the first digit must be 1 and repetition of the digits is
allowed?

Solution:

One (1) choice for the first digit. Four (4) choices for the last 3
digits that form the 4 digit number since repetition is allowed. Hence the
number N of numbers that we may form is given by:

N=1 × 4 × 4 × 4=64

The total number of numbers that we may form is 64.


10
9. A coin is tossed three times. What is the total number of all
possible outcomes?

Solution:

The first time the coin is tossed, 2 different outcomes are possible
(heads, tails). The second time the coin is tossed, another 2 different
outcomes are possible and the third time the coin is tossed, another 2
different outcomes are possible. Hence the total number of possible
outcomes is equal to:

N=2× 2× 2=8

There are 8 total number of possible outcomes.

10. In a certain country telephone numbers have 9 digits. The first


two digits are the area code (03) and are the same within a
given area. The last 7 digits are the local number and cannot
begin with 0. How many different telephone numbers are
possible within a given area code in this country?

Solution:

The first digit of the area code is 0, no choice which is in fact one
choice only. The second digit of the area code is 1, no choice or one
choice only. The first digit of the local code can be any digit except 0,
so 9 choices. The 2nd, 3rd, 4th, 5th, 6th, and 7th digits of the local code can
be any digit, hence 10 choices each.

Using the counting principle, the total number of possible


telephone numbers is given by:

N=1 ×1 × 9× 10× 10 ×10 ×10 ×10 ×10=9 000 000

Therefore, the total number of possible telephone numbers is


9 000 000.

11. In a school, ID cards have 5 digit numbers. How many ID cards


can be formed if repetition of the digit is not allowed?

Solution:

The first digit of the number to be formed can be any of the 10


digits, hence the 10 choices. The second digit can be any of the 10
digits except the digit used in position 1 since no repetition of the digits
is allowed, hence 9 choices. The third digit can be any of the 10 digits
excepts the two already used in positions 1 and 2 since repetition is not
allowed, hence 8 choices and so on.
11
The number N of ID cards is given by:

N=10 × 9 ×8 ×7 × 6=30 240

Thus, the number of ID cards is 30 240.

12. Using the digits 1, 2, 3, and 5, how many 4 digit numbers can
be formed if the first digit must be 1 and repetition of the digits
is not allowed?

Solution:

One (1) choice for the first digit. 3 choices for the second digit of
the number to be formed since repetition is not allowed. 2 choices for
the third digit of the number to be formed. 1 choice for the fourth digit of
the number to be formed. Hence the number N of numbers that we may
form is given by:

N=1 ×3 × 2× 1=6

The number of numbers that we may form is 6.

13. Two dice are rolled. What is the total number of all possible
outcomes?

Solution:

Six possible outcomes for the first die (1, 2, 3, 4, 5, 6) and 6 other
possible outcomes for the second die. The total number of different
outcomes is:

N=6 ×6=36

The total number of different outcomes is 36.

14. A coin is tossed and a die is rolled. What is the total number off
all possible outcomes?

Solution:

Two possible outcomes for the coin (heads, tails) and 6 possible
outcomes (1,2,3,4,5,6) for the die. The total number of different
outcomes is:

N=2× 6=12

The total number of different outcomes is 12.


12
15. Using the digits 1, 2, 3, and 5, how many 4 digit numbers can
be formed if the number must be divisible by 2 and repetition is
allowed?

Solution:

For the number to be formed to be divisible by two, the last digit


must be 2, hence one choice for this digit. 4 choices for each of the
other digits since repetition is allowed. Hence the number N of numbers
that we may form is given by:

N=4 × 4 × 4 × 1=64

Therefore, the number N of numbers that we may form is 64.

I think you already understand the practice activity above. To


check your understanding, answer the activity below.

Activity 4: Independent Practice

Answer the following problems solving below. Show your solution


to other sheet.

1. A restaurant offers four sizes of pizza, two types of crust, and eight
toppings. How many possible combination of pizza with one topping
are there?

2. How many ways can 5 paintings be line up on the wall?

3. Rod has 4 shirts, 3 pairs of pants, and 2 pairs of shoes that all
coordinate. How many outfits can you put together?

4. In how many ways can 6 bicycles be parked in a row?

5. How many 2 digit numbers can you make using the digits 1, 2, 3, and
4 without repeating the digits?

6. Anne loves to eat salad! How many salads can she put together if
she
can choose one from each of the following: two types of lettuce, 4
types of vegetables and 7 types of dressing?

7. Suppose most of your clothes are dirty and you are left with 2 pants
and 3 shirts. How many choices do you have or how many different
ways can you dress?
13
8. You go a restaurant to get some breakfast. The menu says
pancakes,
waffles, or home fries. And for drink, coffee, juice, hot chocolate,
and
tea. How many different choices of food and drink do you have?

9. You are buying a new car. There are two body style (sedan or
hatchback).There are 5 colors available (black, red, green, blue, and
white). And there are 3 models (GL, SS, and SL). How many total of
choices are there?

10. If five digits 1, 2, 3, 4, 5 are being given and a three digit code has
to
be made from it if the repetition of digits is allowed then how many
such codes can be formed.

11. A man has 5 different suits, 3 different shirts and 5 different pairs
of shoes. In how many ways can this man wear a suit, a shirt and
a pair of shoes?

12. There are eight finalist in the Miss Philippines contest. How many
different outcomes are possible if one person will be selected First
Runner-Up and another will be Miss Philippines?

13. A seven-question quiz has four true/false questions followed by 3


multiple choice questions. For each multiple choice questions there
are four possible answer. In how many different ways is it possible
to
answer the seven questions?

14. Reymark, Marvin, John, Paul, Juren, and Johann are running a
race.
How many different orders of finish are possible?

15. Ronnie is planning a date with his girlfriend. He plans for them to
watch a movie, go out to dinner, and then attend a sporting event.
He is deciding between 5 movies, 8 restaurants, and 2 sporting
events. How many different dates can Ronnie plan?

I hope you answered all of the item! To check your score!


If your score is:

11-15 You are doing great! You may proceed to the next
activity, the Assessment (Activity 8) after reviewing the
missed items.
6-10 You are doing well but you need to proceed to the next
activities/exercises of this lesson.
1-5 You need to study harder. Ask help from your teacher or
parent or friend. Study the next examples and answer the
next activities.

14

What I Have Learned


Activity 5: Accomplish With Me!

Answer the following problems solving. Show your solution.

1. If Zian has 10 T-shirts, 7 pairs of pants, and 4 pairs of shoes, how


many possibilities can he dress himself up for the day?

2. In how many ways can 9 children arrange themselves in a row for a


picture taking?

3. A dress-shop owner has 15 new dresses that she wants to display in


the window if the display window has 5 mannequins, in how many
ways can she dress them up?

4. How many four-digit numbers can be formed from the numbers 1, 3,


4, 6, 8, and 9 if repetition of digits is not allowed?

5. From among the 36 teachers in a college, one principal, one vice-


principal and the teacher-in charge are to be appointed. In how
many ways this can be done?

What I Can Do
Activity 6: I Can Do It!
Perform the activity as indicated. Show your solution to the other
sheets given.

1. There are four Gators in a holding cell at the jail. They will be asked
to arrange themselves from left to right in a police line-up. How many
different line-ups are possible?

15

2. Given 7 flags of different colors, how many different signals can


be generated
if a signal requires the use of two flags, one below the other?

3. You have 10 pairs of pants, 6 shirts, and 3 jackets. How many outfits
can you have consisting of a shirt, a pair of pants, and a jacket?

4. How many different ways are there to arrange your first three classes
if they are math, science, and language arts?

5. Twelve students compete in a race. In how many ways first three prizes


be
given?

6. You take a survey with five “yes” or “no” answers. How many
different
ways could you complete the survey?

7. There are 6 people who will sit in a row but out of them Ronnie will
always be left of Annie and Rachel will always be right of Annie. In
how many such arrangement can be done?

8. In how many ways 6 children can be arranged in a line if two


particular children of them are always together?

9. 10 students have appeared in a test in which the top three will get a
prize. How many possible ways are there to get the prize winners?

10. It is required to seat 5 men and 4 women in a row so that the


women occupy the even places. How many such arrangements are
possible?
16

Assessment
Activity 7: Guided Assessment

Study the different problems with solution below. If there are


some item that you don’t understand, do not hesitate to ask assistance
from your teacher’s, parent, and friend.

1. Ernie has 5 tops, 6 skirts and 4 caps from which to choose an outfit.
In how many ways can she select one top, one skirt and one cap?

Solution:

Ways=5 ×6 × 4=120

2. Coldstone Creamery has 10 different choices of ice cream, 12


different
choices of toppings, and 3 different sizes of waffle bowls. If Edward
wants to get a waffle bowl, one kind of ice cream, and one topping,
how many different ways can he choose his dessert?

Solution:

There are:
10 = ice cream
12 = toppings
3 = waffle bowl

So, the number N of different ways that Edward choose his


dessert is,

N=10 × 12× 3=360different ways.

3. To buy a computer system, a customer can choose one of 4


monitors, one of 2 keyboards, one of 4 computers and one of 3
printers. Determine the number of possible systems that a
customer can choose from.

Solution:

Using the counting principle used in the introduction above, the


number of all possible computer systems that can be bought is given
by:

N=4 × 2× 4 × 3=96

17

Therefore, the number of possible systems that a customer can


be choose from is 96.

4. A student can select one of 6 different mathematics books, one


of three different chemistry books and one of 4 different science
books. In how many different ways can a student select a book of
mathematics, a book of chemistry and a book of science?

Solution:

The total number N of different ways that the students can select
his 3 books is given by:

N=6 ×3 × 4=72

So, the number N of different ways that the students can select
his 3 books is 72.

5. Tarryn has five different skirts, four different tops and three pairs of
shoes. Assuming that all the colours complement each other, how
many different outfits can she put together?

Solution:

The total number N of different outfits that Tarryn’s can be put


together is:

N=5 × 4 × 3=60 different outfits.

Thus, the total number N of different outfits is 60.

6. A man has 3 different suits, 4 different shirts and 5 different pairs


of shoes. In how many ways can this man wear a suit, a shirt and
a pair of shoes?

Solution:
The total number N of different ways that this man can wear one
of his suits, one of his shirts and a pair of his shoes is:

N=3× 4 × 5=60

Therefore, the total number of different ways that this man can
wear one of his suits, one of his shirts and a pair of his shoes is 60.

7. The school cafeteria offers five sandwich choices, four desserts, and
three beverages. How many different meals consisting of one
sandwich, one dessert, and one beverage can be ordered?

18
Solution:

The choices are:

5 = sandwich
4 = desserts
3 = beverages

N=5 × 4 × 3=60

Hence, there are 60 different meals consisting of one sandwich,


one dessert, and one beverage that can be ordered.

8. Using the digits 1, 2, 3, and 5, how many 4 digit numbers can be


formed if the first digit must be 1 and repetition of the digits is
allowed?

Solution:

One (1) choice for the first digit. Four (4) choices for the last 3
digits that form the 4 digit number since repetition is allowed. Hence the
number N of numbers that we may form is given by:

N=1 × 4 × 4 × 4=64

The total number of numbers that we may form is 64.

9. A coin is tossed three times. What is the total number of all


possible outcomes?

Solution:

The first time the coin is tossed, 2 different outcomes are possible
(heads, tails). The second time the coin is tossed, another 2 different
outcomes are possible and the third time the coin is tossed, another 2
different outcomes are possible. Hence the total number of possible
outcomes is equal to:
N=2× 2× 2=8

There are 8 total number of possible outcomes.

10. In a certain country telephone numbers have 9 digits. The first


two digits are the area code (03) and are the same within a
given area. The last 7 digits are the local number and cannot
begin with 0. How many different telephone numbers are
possible within a given area code in this country?

19
Solution:

The first digit of the area code is 0, no choice which is in fact one
choice only. The second digit of the area code is 1, no choice or one
choice only. The first digit of the local code can be any digit except 0,
so 9 choices. The 2nd, 3rd, 4th, 5th, 6th, and 7th digits of the local code can
be any digit, hence 10 choices each.

Using the counting principle, the total number of possible


telephone numbers is given by:

N=1 ×1 × 9× 10× 10 ×10 ×10 ×10 ×10=9 000 000

Therefore, the total number of possible telephone numbers is


9 000 000.

11. In a school, ID cards have 5 digit numbers. How many ID cards


can be formed if repetition of the digit is not allowed?

Solution:

The first digit of the number to be formed can be any of the 10


digits, hence the 10 choices. The second digit can be any of the 10
digits except the digit used in position 1 since no repetition of the digits
is allowed, hence 9 choices. The third digit can be any of the 10 digits
excepts the two already used in positions 1 and 2 since repetition is not
allowed, hence 8 choices and so on.

The number N of ID cards is given by:

N=10 × 9 ×8 ×7 × 6=30 240

Thus, the number of ID cards is 30 240.

12. A school plays a series of 6 soccer matches. For each match there
are 3 possibilities: a win, a draw or a loss. How many possible
results
are there for the series?

Solution:

Step 1: Determine how many outcomes you have to choose from for
each event

There are 3 outcomes for each match: win, draw or lose.

20
Step 2: Determine the number of events

There are 6 matches, therefore the number of events is 6.

Step 3: Determine the possible outcomes

There are 3 possible outcomes for each of the 6 events.


Therefore, the total number of possible outcomes for the series of
matches is:

N=3× 3 ×3 ×3 ×3 ×3=36 =729.

13. Two dice are rolled. What is the total number of all possible
outcomes?

Solution:

Six possible outcomes for the first die (1, 2, 3, 4, 5, 6) and 6 other
possible outcomes for the second die. The total number of different
outcomes is:

N=6 ×6=36

The total number of different outcomes is 36.

14. A coin is tossed and a die is rolled. What is the total number off
all possible outcomes?

Solution:

Two possible outcomes for the coin (heads, tails) and 6 possible
outcomes (1,2,3,4,5,6) for the die. The total number of different
outcomes is:

N=2× 6=12
The total number of different outcomes is 12.

15. A gift basket is made up from one CD, one book, one box of
sweets,
one packet of nuts and one bottle of fruit juice. The person who
makes up the gift basket can choose from five different CD’s, eight
different books, three different boxes of sweets, four kinds of nuts
and six flavours of fruit juice. How many different gift baskets can
be
produced?

21
Solution:

5 = CD’s
8 = Books
3 = boxes of sweets
4 = nuts
6 = flavours of fruit juice

N=5 × 8 ×3 × 4 ×6=2 880possible gifts

Therefore, the number N of different gift basket is 2 880.

I think you already understand the practice activity above. To


check your understanding, answer the activity below.

Activity 8: Independent Assessment

Answer the following problems solving. Show your solution to the


separate sheet.

1. Chloe has 5 shirts, 4 pairs of pants and 4 pairs of sandals. She


needs
to make an outfit containing one of each item. How many different
outfits are possible?

2. Kenny is going to purchase a new car. The car has 5 exterior color
choices, 2 interior color packages and 2 engine options. How many
different cars are possible?

3. A movie theatre sells 3 sizes of popcorn (small, medium, and large)


with 3 choices of toppings (no butter, butter, extra butter). How many
possible ways can a bag of popcorn be purchased?
4. Angel’s Burger offers 4 types of burgers, 5 types of beverages, and 3
types of dessert, how many possible meals can be chosen?

5. Maggie is choosing silverware with which to eat dinner. There are 3


knives and 4 forks to choose from. How many different silverware
sets
can Maggie choose?

6. Ron is ordering a birthday cake for a friend. There are 5 cake flavors
and 3 frosting flavors to choose from. How many different cakes can
Ron order?

7. A men’s department store sells 3 different suit jackets, 6 different


shirts, 8 different ties, and 4 different pairs of pants. How many
different suits consisting of a jacket, shirt, tie, and pants are
possible?

22
8. A baseball manager is determining the batting order for the team.
The
team has 9 players, but the manager definitely wants the pitcher to
bat
last. How many batting orders are possible?

9. How many eight-digit numbers can be formed if the leading digit


cannot be a zero and the last number cannot be 1?

10. The standard configuration for an Alaska license plate is 3 letters


followed by 3 digits. How many different license plate are possible if
letters and digits can be repeated?

11. How many 3-letter code words can be formed if at least one of the
letters is to be chosen from the vowels a, e, i, o, and u?

12. How many license plates of 3 symbols (letters and digits) can be
made using at least 2 letters for each?

13. In how many different ways can a10-question true-false test be


answered if every question must be answered?

14. In how many different ways can a10-question true-false test be


answered if it is all right to leave questions unanswered?

15. You are taking a survey on your experience at Taco Bell. For the
first
five questions you can answer Below Average, Average, or Above
Average for each question. The last three questions you can
respond
with either Agree or Disagree. How many total outcomes are there
for this survey?

Thank you for answering this activity. To check your score!


If your score is:

11-15 You are doing great! You may proceed to lesson 2 after
reviewing the missed items.
6-10 You are doing well but you need to proceed to the next
activities/exercises of this lesson.
1-5 You need to study harder. Ask help from your teacher or
parent or friend. Study the next examples and answer the
next activities.

23

Additional Activities
Answer the following problems solving. Show your solution to the
separate sheet.

1. A computer password is consist of two lower case letters followed by


four digits. Determine how many passwords are possible if repetition
of letters and digits is permitted?

2. A computer password is consist of two lower case letters followed by


four digits. Determine how many passwords are possible if repetition
of letters and digits is NOT permitted?

3. Liza, Michelle, Sally, and Bob are waiting in line to buy concert
tickets.
In how many different ways can they stand in line?

4. The daily double at most racetracks consist of selecting the winning


horse in both the first and the second race. If the first race has 7
entries
and the second has 8 entries, how many daily double tickets must
you
purchase to guarantee a win?
5. A class ring has three stone options (emerald, ruby or sapphire), two
metal options (gold and silver). How many different class rings are
possible?

6. If the NCAA has application from 6 universities for hosting its


intercollegiate tennis championships in 20008 and 2009, how many
ways may they select the hosts for these championships if they are
not both to be held at the same university.

7. There are six finalists in the Mr. Rock Hill pageant. In how many
ways
may the judges choose a winner and the first runner-up?

8. Find the total number of outcomes when choosing three different


bike
colors and two different styles?

9. You want to take a class at Bucks over the summer. You can only
take
a class on Monday or Friday. The classes offered those days are
Dance, Painting and Math. Determine the number of possibilities for
classes.

10. A golf club manufacturer makes irons with 7 different shaft lengths,
3 different grips, 5 different lies, and 2 different club head materials.
How many different combinations are offered?

24

Solving Problems
using the
Lesso
Permutations
n Formula
2
What I Need to Know
This lesson is written for you to solve problems in permutations of
which the number of permutations of n objects taken r at a time is:
n!
P ( n , r )= , n ≥r . And the permutations of n objects taken all the
( n−r ) !
time is: P ( n , n )=n !. To solve different level of problems, you are going
to use the permutations formula.

You may SKIP this lesson or its activities if you already master the skill
and get a score of 11 out of 15 items in the pre-test.

What I Know
Find out how much you already know about this lesson. Choose
the letter that you think is correct and write your answer on the space
provided before each number on the separate sheet. Take note of the
items that you were not able to answer correctly and look for the right
answer as you go through this module.

___1. There are 4 czech and 3 slovak books on the bookshelf. Czech
books should be placed on the left side of the bookshelf and
slovak books on the right side of the bookshelf. How many ways
are there to arrange the books?
A. 140 B. 142 C. 144 D. 148

___2. How many five-digit numbers can be formed from the numbers
1, 2, 3, 4, 6, 7, 8, and 9 if repetition of digits is not allowed?
A. 6 000 B. 6 500 C. 6 662 D. 6 720

25
___3. A zip code contains 5 digits. How many different zip codes can
be made with the digits 0–9 if no digit is used more than once
and the first digit is not 0?
A. 27 216 B. 27 220 C. 28 216 D. 28 220

___4. In how many different ways can 11 people occupy the 11 seats in
a back row of a mini-theater?
A. 38 900 880 B. 38 916 880
C. 39 900 800 D. 39 916 800

___5. A doctor wants to assign 5 different tasks to his 5 nurses. In how


many possible ways can he do it?
A. 120 B. 240 C. 360 D. 420

___6. In a town fiesta singing competition with 10 contestants, in how


many can the organizer arrange the first three singers?
A. 620 B. 700 C. 720 D. 800

___7. In how many ways 6 children can be arranged in a line, if t wo


particular children of them are never together?
A. 450 B. 480 C. 550 D. 580

___8. Determine the different ways can 9 potted plants be arranged in


a row?
A. 361 800 B. 360 880 C. 362 800 D. 362 880

___9. A licence plate begins with three letters. If the possible letters are
A, B, C, D and E, how many different permutations of these
letters can be made if no letter is used more than once?
A. 30 B. 40 C. 60 D. 70

___10. In how many ways can you place 9 different books on a shelf if
there is space enough for only 4 books?
A. 3 000 B. 3 024 C. 4 000 D. 4 024

___11. If a class has 28 students, how many different arrangements


can 5 students give a presentation to the class?
A. 11 793 600 B. 11 800 601
C. 12 793 600 B. 12 800 601

___12. A dress-shop owner has 10 new dresses that she wants to


display in the window. If the display window has 6 mannequins,
in how many ways can she dress them up?
A. 150 000 B. 150 200 C. 151 000 D. 151 200

26
___13. It is required to seat 4 men and 4 women in a row so that the
women occupy the even places. How many such arrangements
are possible?
A. 500 B. 550 C. 576 D. 600

___14. Consider a set having 5 elements L, M, N, O, and P. In how


many ways 3 elements can be selected out of the total number
of elements if repetition is not allowed?
A. 15 B. 30 C. 45 D. 60

___15. Sixteen skiers are competing in the final round of Olympic


freestyle skiing aerial competition. In how many different ways
can 3 of the skiers finish first, second, and third to win the gold,
silver, and bronze medals?
A. 3 360 B. 3 370 C. 4 360 D. 4 370

Great job! You are done answering. And now, kindly check your
score! If your score is:
11-15 You are doing great! You may proceed to the next
activity that can be found in What’s More after
reviewing the missed items.
6-10 You are doing well but you need to proceed to the next
activities/exercises of this lesson.
1-5 You need to study harder. Ask help from your teacher
or parent or friend. Study the next examples and
answer the different activities.

What’s In

You were done in lesson 1, and now you are studying lesson 2 of
this module. I know you can simply understand this lesson because in
understanding permutations, you should have the skills and knowledge
in the basic counting principle called the fundamental counting principle
that you learned in lesson 1.

27
What’s New
Activity 10: Can You Show Me the Formula?

1. In how many ways can four-letter word CODE be arrange its letters 3
at a time?

Solution: Using the permutation formula

The problem involves 4-letter word CODE taken 3 at a time.

4! 4 ! 4 ×3 × 2× 1
P( 4,3)= = = =4 × 3× 2=24
( 4−3 ) ? 1 ! 1

There are 24 possible ways of arranging the 4-letter word CODE.

2. In how many ways can 4 flower vase wish to arrange 4 of them in a row for
the birthday party?

Solution: Using the permutation formula

The problem involves 4 flower vase wish to arrange 4 of them in a


row.

P( 4,4)=4 ×3 ×2 ×1=24

Therefore, there are 24 possible ways.

In this example, we used all the numbers from n=4 down to 1.


Another way of writing
4 ×3 ×2 ×1 is 4! (read as 4 factorial).
Similarly, 5 !=5× 4 × 3 ×2 ×1=120
6 !=6 ×5 × 4 ×3 × 2× 1=720
3 !=3× 2× 1=6
1 !=1

28
For convenience, we define 0 !=1
Thus, in example 2, P ( 4,4 )=4 !=24

3. In how many ways can a president, a treasurer and a secretary be


chosen from among 7 candidates?

Solution: Using permutation formula

The problem involves 7 candidates taken 3 at a time.

7! 7 ! 7 ×6 × 5× 4 × 3 ×2 ×1
P(7,3)= = = =7 ×6 × 5=210
( 7−3 ) ! 4 ! 4 ×3 × 2× 1

There are 210 possible ways to choose a president, a treasurer


and a secretary be chosen from among 7 candidates.

Activity 11: Let’s Try This!

Answer each permutations completely: Show your solution to the


other sheets given.

1. A license plate begins with three letters. If the possible letters are J, K, L, M
and N, how many different permutations of these letters can be made if no
letter is used more than once?

2. In how many ways can 7 people arrange themselves in a row for picture
taking?

3. In a town fiesta singing competition with 10 contestants, in how many


can the organizer arrange the first three singers?

4. Determine the different ways can 9 potted plants be arranged in a


row?

29
5. In how many different ways can 8 different-colored goats be
positioned in a carousel?

6. A doctor wants to assign 5 different tasks to his 5 nurses. In how


many possible ways can he do it?

7. In how many ways can 5 cars be parked if there are 9 available


parking spaces?

8. In how many different ways can 11 people occupy the 11 seats in a


back row of a mini-theater?

9. In how many ways can you place 9 different books on a shelf if there
is space enough for only 4 books?

10. Twelve runners join a racing competition. In how many ways can
they be arranged as first, second, and third placers?

What Is It
What is Permutation?
A permutation is an arrangement of all or part of a set of
objects, with regard to the order of the arrangement.

For example:
1. Suppose we have a set of three letters: A, B, and C. We might ask
how many ways we can arrange 2 letters from that set. Each
possible arrangement would be an example of a permutation. The
complete list of possible permutations would be: AB, AC, BA, BC,
CA, and CB.

30
When they refer to permutations, statisticians use a specific
terminology. They describe permutations as n distinct objects taken r at
a time. Translation: n refers to the number of objects from which the
permutation is formed; and r refers to the number of objects used to
form the permutation. Consider the example from the previous
paragraph. The permutation was formed from 3 letters (A, B, and C),
so n = 3; and the permutation consisted of 2 letters, so r = 2.

Computing the number of Permutations


The number of permutations of n objects taken r at a time is
n Pr = n(n - 1)(n - 2) ... (n - r + 1) = n! / (n - r)!
or
n!
P ( n , r )= , n ≥r , The Formula
( n−r ) !

Solution: Using permutation formula

The problem involves 3 letters taken 2 at a time.

3! 3 ! 3× 2× 1
P(3,2)= = = =3 ×2=6
( 3−2 ) ! 1 ! 1

Thus, there 6 possible arrangement of the set of three letters (A, B, C).

2. How many 3 letter words with or without meaning can be formed out
of the letters of the word SWING when repetition of letters is not
allowed?

Solution: 

Here n = 5, as the word SWING has 5 letters. Since we have to


frame 3 letter words with or without meaning and without repetition,
therefore total permutations possible are:

Using permutation formula

The problem involves 5 letters taken 3 at a time.

5! 5 ! 5 × 4 ×3 ×2 ×1
P(5,3)= = = =5 × 4 ×3=60
( 5−3 ) ! 2 ! 2× 1

So, there 60 letters that can be formed of the word SWING with or
without meaning.

31
What’s More
Activity 12: Guided/Controlled Practice

Study the practice below.

1. If three alphabets are to be chosen from A, B, C, D and E such that


repetition is not allowed then in how many ways it can be done?

Solution:

The number of ways three alphabets can be chosen from five will
be,

5! 5 × 4 ×3 ×2 ×1
P ( 5,3 )= = =60
( 5−3 ) ! 2× 2

Hence, there are 60 possible ways.

2. 10 students have appeared in a test in which the top three will get a
prize. How many possible ways are there to get the prize winners?

Solution:

We need to choose and arrange 3 persons out of 10. Hence the


number of possible ways will be,

10 ! 10 ×9 × 8× 7 ×6 ×5 × 4 × 3× 2× 1
P ( 10,3 )= =
( 10−3 ) ! 7 × 6 ×5 × 4 ×3 ×2 ×1

¿ 10 ×9 × 8=720

3. In how many ways the word HOLIDAY can be rearrange such that
the
letter I will always come to the left of letter L?

Solution:

Number of letters in HOLIDAY = 7 and there is no repetition of


letters. Hence, the number of ways all letters can be arranged is 7!.

The number of ways the letters are arranged such that I will come
7!
left of L will be, as in half of the arrangements L will be right of I and
2
in other half it will be on left of I.

7 ! 7 ×6 × 5× 4 × 3 ×2 ×1 5 040
P= = = =2520
2 2 2
32
Thus, the number of letters in HOLIDAY if the letter I will always
come to the left of letter L is 2 520.

4. There are 6 people who will sit in a row but out of them Ronnie will
always be left of Annie and Rachel will always be right of Annie. In
how many ways such arrangement can be done?

Solution:

The total number of ways of 6 people being seated in a row will


be 6! Now, with the given constraint the total number of ways will be
6!
.
3!
Thus,

6 ! 6 ×5 × 4 × 3× 2× 1
P= = =6 ×5 × 4=120
3! 3 ×2 ×1

It implies that out of 6 people arrangement of arrangement of 3


people is predefined.

5. In how many ways can 4 different resistors be arranged in series?

Solution:

Since there are 4 objects, the number of ways is:

P=4 !=4 ×3 ×2 ×1=24 ways

Thus, there are 24 ways that 4 different resistors be arranged.

6. How many different number-plates for cars can be made if each


number-plate contains four of the digits 0 to 9 followed by a letter A
to
Z, assuming that no repetition is allowed?

Solution:

There are 10 possible digits (0, 1, 2, . . ., 9) and we need to take


them 4 at a time. There are 26 letters in the alphabet.

With no repetition, we have:

10! 10 ! 10 ×9 × 8× 7 ×6 ×5 × 4 × 3× 2× 1
P ( 10,4 ) ×26= × 26= × 26= ×26=10 × 9× 8 ×7 ×26=
( 10−4 ) ! 6! 6 ×5 × 4 ×3 ×2 ×1

33
7. How many different number-plates for cars can be made if each
number-plate contains four of the digits 0 to 9 followed by a letter A
to
Z, assuming that repetition is allowed?

Solution:

With repetition, we have simply:

(number of digits from 0000 to 9999) x 26

P=10 000 × 26=26 000.

8. How many 3 letter words can we make with the letters on the word
LOVE?

Solution:

There are 4 letters in the word love and making 3 letter words is
similar to arranging these 3 letters and order is important since LOV
and VOL are different words because of the order of the same letters L,
O and V. Hence it is a permutation problem. The number of word given
by,

4! 4!
P ( 4,3 ) = = =4 ×3 × 2× 1=24.
( 4−3 ) ! 1

Therefore, there are 24 different ways can we make with the


letters on the word LOVE.

9. How many 3 digit numbers can we make using the digits 2, 3, 4, 5,


and 6 without repetitions?

Solution:

There are 5 digit numbers. Without repetitions, we have simply:

5! 5 ! 5 ×4 ×3 ×2 ×1
P ( 5,3 )= = = =5 × 4 ×3=60
( 5−3 ) ! 2 ! 2×1

So, there are 60 ways can we make of 3 digit numbers using the
digits 2, 3, 4, 5, and 6.

10. How many 6 letter words can we make using the letters in the word
LIBERTY without repetitions?

Solution:

There are 7 letters in the word LIBERTY and making 6 letter


words without repetition.
34
n!
We will use the formula in solving: P ( n , r )= . So we simply
( n−r ) !
have:

7! 7!
P ( 7,6 )= = =7 ×6 × 5× 4 × 3 ×2 ×1=5 040.
( 7−6 ) ! 1!

11. In how many ways can a supermarket manager display 5 brands of


cereals in 3 spaces on a shelf?

Solution:

This is asking for the number of permutations, since we don’t


want repetitions. The number of ways is:

5! 5 ! 5 ×4 ×3 ×2 ×1
P ( 5,3 )= = = =5 × 4 ×3=60.
( 5−3 ) ! 2 ! 2×1

12. Find the number of ways to arrange 6 items in groups of 4 at a time


where order matters.

Solution:

6! 6 ! 6 ×5 × 4 ×3 ×2 ×1 720
P ( 6,4 ) = = = = =360
( 6−4 ) ! 2! 2×1 2

There are 360 possible ways to arrange 6 items in groups of 4 at


a time.

13. How many different ways can the offices of president, vice
president,
secretary, treasurer, and auditor be chosen from an organization of
50 members?

Solution:

There are 50 members in an organization, then we choose a


president, vice-president, secretary, treasurer, and auditor.

50 ! 50 !
P ( 50,5 )= = =50× 49 × 48 ×47 × 46=254 251 200
( 50−5 ) ! 45 !

Thus, the different ways can the offices of president, vice


president, secretary, treasurer, and auditor be chosen from an
organization of 50 members is 254 251 200.
35
14. If a class has 28 students, how many different arrangements can 5
students give a presentation to the class?

Solution:

Using the permutations formula which the n=28 and the r =5.

28! 28 !
P ( 28,5 )= = =28 × 27 ×26 ×25 ×24=11 793 600
( 28−5 ) ! 23 !

Therefore, there are 11 793 600 different arrangements.

15. John has 10 marbles in his bags. In how many ways can he pick 6
marbles from the bag?

Solution:

10! 10 ! 10 ×9 × 8 ×7 ×6 × 5× 4 × 3 ×2 ×1
P ( 10,6 )= = = =10 ×9 × 8× 7 ×6 ×5=151 200
( 10−6 ) ! 4 ! 4 ×3 ×2 ×1

Thus, there are 151 200 possible ways can he pick 6 out of 10
marbles from the bag.

You are done studying the activity above, to check if you get this,
please answer the next activity.

Activity 13: Independent Practice

Answer each problems completely: Show your solution on the


sheets provided.

1. Suppose that 7 people enter a swim meet. Assuming that there are
no
ties, in how many ways could the gold, silver, and bronze medals be
awarded?

2. A coach must choose how to line up his five starters from a team of
12 players. How many different ways can the coach choose the
starters?

3. How many different four-letter password can be created for a


software
access if no letter can be used more than once?
36
4. How many different ways you can elect a Chairman and Co-
Chairman
of a committee if you have 10 people to choose from.

5. There are 25 people who worked in an office together. Five of these


are selected to attend five different conferences. The first person
selected will go to a conference in a Hawaii, the second will go New
York, the third will go to San Diego, the fourth will go to Atlanta, and
the fifth will go to Nashville. How many such selections are possible?

6. Jhon couldn’t recall the Serial number on his expensive bicycle. He


remembered that there were 6 different digits, none used more than
once, but couldn’t remember what digits were used. He decide to
write
down all of the possible 6 digit numbers. How many different
possibilities will he have to create?

7. One hundred twelve people bought raffle tickets to enter a random


drawing for three prizes. How many ways can three names be drawn
for first prize, second prize, and third prize?

8. A disc jockey has to choose three songs for the last few minutes of
his
evening show. If there are nine songs that he feels are appropriate for
that time slot, then how many ways can he choose and arrange to
play
three of those nine songs?

9. If four alphabets are to be chosen from Q, R, S, T, U, and V such


that
repetition is not allowed then in how many ways it can be done?

10. 12 students have appeared in the exam in which the top four will
get
a prize. How many possible ways are there to get the prize
winners?

11. How many 3 letter words can we make with the letters on the word
SOLVE?

12. How many ways can the letters of the word PARK be arranged?

13. In how many ways can 4 of 7 different kinds of bushes be planted


along a walkway?

14. A club with 10 members is to choose 3 officers-president, vice-


president and secretary. How many ways can these offices be
filled?
15. Suppose you are asked to list, in order of preference, the three best
movies you have seen this year. If you saw 20 movies, in how
many
ways can the 3 best be chosen and ranked?

37

Nice work! Since you answered this activity. Check your score!
If your score is:

11-15 You are doing great! You may proceed to the next
activity, the Assessment (Activity 17) after reviewing the
missed items.
6-10 You are doing well but you need to proceed to the next
activities/exercises of this lesson.
1-5 You need to study harder. Ask help from your teacher or
parent or friend. Study the next examples and answer the
next activities.

What I Have Learned


Activity 14: Warm That Brain Up!

Answer the following problems solving. Show your solution to the


separate sheet.

1. Sixteen skiers are competing in the final round of the Olympic


freestyle skiing aerial competition. In how many different ways can 3
of the skiers finish first, second, and third to win the gold, silver, and
bronze medals?

2. In how many ways can 9 children arrange themselves in a row for a


picture taking?
3. A dress-shop owner has 10 new dresses that she wants to display in
the window. If the display window has 6 mannequins, in how many
ways can she dress them up?

4. How many five-digit numbers can be formed from the numbers


1, 2, 3, 4, 6, 7, 8, and 9 if repetition of digits is not allowed?

38
5. From among the 36 teachers in a college, one principal, one vice-
principal and the teacher-in charge are to be appointed. In how
many ways this can be done?

What I Can Do
Activity 15: Tell Me If I Can Do It?

Perform the activity as indicated. Show your solution to the


separate paper.

1. Given 7 different flags, in how many ways can be arrange if two flags
use below the other?

2. A professor wants to write and ordered 7-questions test from a pool


of 10 questions. How many different forms of the test can the
professor write?

3. How many positive integers of 5 digits may be made from the ciphers
1, 2, 3, 4, 5, if each cipher may be used just once? 
4. There are 4 czech and 3 slovak books on the bookshelf. Czech books
should be placed on the left side of the bookshelf and slovak books on the
right side of the bookshelf. How many ways are there to arrange the
books?

5. There are 4 elements a, b, c, and d. Use them to make permutations


(without repetition).

6. Seven athletes are participating in a race. In how many ways can the
first three prizes be won?

7. How many 3 letter words with or without meaning can be formed out
of the letters of the word SOLUTION when repetition of words is
NOT allowed?

39

8. In how many ways 6 children can be arranged in a line, if t wo


particular children of them are never together?

9. Consider a set having 5 elements R,S,T,U,V. In how many ways 3


elements can be selected (without repetition) out of the total number
of elements.

10. It is required to seat 4 men and 4 women in a row so that the


women occupy the even places. How many such arrangements are
possible?

Assessment
Activity 16: Guided Assessment

Study and understand the different problems below.

1. If three alphabets are to be chosen from A, B, C, D and E such that


repetition is not allowed then in how many ways it can be done?

Solution:

The number of ways three alphabets can be chosen from five will
be,

5! 5 × 4 ×3 ×2 ×1
P ( 5,3 )= = =60
( 5−3 ) ! 2× 2

Hence, there are 60 possible ways.


2. 10 students have appeared in a test in which the top three will get a
prize. How many possible ways are there to get the prize winners?

Solution:

We need to choose and arrange 3 persons out of 10. Hence the


number of possible ways will be,

10 ! 10 ×9 × 8× 7 ×6 ×5 × 4 × 3× 2× 1
P ( 10,3 )= =
( 10−3 ) ! 7 × 6 ×5 × 4 ×3 ×2 ×1

¿ 10 ×9 × 8=720

40
3. Twenty bands have applied to march in the parade, but only seven
spots are available. How many permutations of seven bands are
possible for their order in the parade?

Solution:

There are 20 bands applied to march in the parade but only 7


spots are available. We simply ha:

20 ! 20!
P= = =20 ×19 ×18 ×17 ×16 × 15× 14=390 700800
( 20−7 ) ! 13!

Thus, there are 390 700 800 possible order in the parade.

4. There are 6 people who will sit in a row but out of them Ronnie will
always be left of Annie and Rachel will always be right of Annie. In
how many ways such arrangement can be done?

Solution:

The total number of ways of 6 people being seated in a row will


be 6! Now, with the given constraint the total number of ways will be
6!
.
3!
Thus,

6 ! 6 ×5 × 4 × 3× 2× 1
P= = =6 ×5 × 4=120
3! 3 ×2 ×1

It implies that out of 6 people arrangement of arrangement of 3


people is predefined.

5. In how many ways can 4 different resistors be arranged in series?


Solution:

Since there are 4 objects, the number of ways is:

P=4 !=4 ×3 ×2 ×1=24 ways

Thus, there are 24 ways that 4 different resistors be arranged.

6. How many different number-plates for cars can be made if each


number-plate contains four of the digits 0 to 9 followed by a letter A
to
Z, assuming that no repetition is allowed?

41
Solution:

There are 10 possible digits (0, 1, 2, . . ., 9) and we need to take


them 4 at a time. There are 26 letters in the alphabet.

With no repetition, we have:

10! 10 ! 10 ×9 × 8× 7 ×6 ×5 × 4 × 3× 2× 1
P ( 10,4 ) ×26= × 26= × 26= ×26=10 × 9× 8 ×7 ×26=
( 10−4 ) ! 6! 6 ×5 × 4 ×3 ×2 ×1

7. How many different number-plates for cars can be made if each


number-plate contains four of the digits 0 to 9 followed by a letter A
to
Z, assuming that repetition is allowed?

Solution:

With repetition, we have simply:

(number of digits from 0000 to 9999) x 26

P=10 000 × 26=26 000.

8. How many 3 letter words can we make with the letters on the word
LOVE?

Solution:

There are 4 letters in the word love and making 3 letter words is
similar to arranging these 3 letters and order is important since LOV
and VOL are different words because of the order of the same letters L,
O and V. Hence it is a permutation problem. The number of word given
by,

4! 4!
P ( 4,3 ) = = =4 ×3 × 2× 1=24.
( 4−3 ) ! 1

Therefore, there are 24 different ways can we make with the


letters on the word LOVE.

9. How many 3 digit numbers can we make using the digits 2, 3, 4, 5,


and 6 without repetitions?

42
Solution:

There are 5 digit numbers.

Without repetitions, we have simply:

5! 5 ! 5 ×4 ×3 ×2 ×1
P ( 5,3 )= = = =5 × 4 ×3=60
( 5−3 ) ! 2 ! 2×1

So, there are 60 ways can we make of 3 digit numbers using the
digits 2, 3, 4, 5, and 6.

10. How many 6 letter words can we make using the letters in the word
LIBERTY without repetitions?

Solution:

There are 7 letters in the word LIBERTY and making 6 letter


words without repetition.
n!
We will use the formula in solving: P ( n , r )= . So we simply
( n−r ) !
have:

7! 7!
P ( 7,6 )= = =7 ×6 × 5× 4 × 3 ×2 ×1=5 040.
( 7−6 ) ! 1!

11. In how many ways can a supermarket manager display 5 brands of


cereals in 3 spaces on a shelf?

Solution:

This is asking for the number of permutations, since we don’t


want repetitions. The number of ways is:
5! 5 ! 5 ×4 ×3 ×2 ×1
P ( 5,3 )= = = =5 × 4 ×3=60.
( 5−3 ) ! 2 ! 2×1

12. Find the number of ways to arrange 6 items in groups of 4 at a time


where order matters.

Solution:

6! 6 ! 6 ×5 × 4 ×3 ×2 ×1 720
P ( 6,4 ) = = = = =360
( 6−4 ) ! 2! 2×1 2

There are 360 possible ways to arrange 6 items in groups of 4 at


a time.

43
13. How many different ways can the offices of president, vice
president,
secretary, treasurer, and auditor be chosen from an organization of
50 members?

Solution:

There are 50 members in an organization, then we choose a


president, vice-president, secretary, treasurer, and auditor.

50 ! 50 !
P ( 50,5 )= = =50× 49 × 48 ×47 × 46=254 251 200
( 50−5 ) ! 45 !

Thus, the different ways can the offices of president, vice


president, secretary, treasurer, and auditor be chosen from an
organization of 50 members is 254 251 200.

14. If a class has 28 students, how many different arrangements can 5


students give a presentation to the class?

Solution:

Using the permutations formula which the n=28 and the r =5.

28! 28 !
P ( 28,5 )= = =28 × 27 ×26 ×25 ×24=11793 600
( 28−5 ) ! 23 !

Therefore, there are 11 793 600 different arrangements.

15. John has 10 marbles in his bags. In how many ways can he pick 6
marbles from the bag?

Solution:
10! 10 ! 10 ×9 × 8 ×7 ×6 × 5× 4 × 3 ×2 ×1
P ( 10,6 )= = = =10 ×9 × 8× 7 ×6 ×5=151 200
( 10−6 ) ! 4 ! 4 ×3 ×2 ×1

Activity 17: Independent Assessment

Answer the following problems below. Show your solution on the


separate sheets given.

1. The manager of a baseball team has named the 9 starters for a game.
He needs to determine the batting order. How many batting orders are
possible?

44
2. There are 33 cars in a car race. The first, second, and third-place
finishers win a prize. How many different arrangements of the first
three
positions are possible?

3. In how many ways is it possible for 8 students to arrange themselves


among 8 seats in the front row of an auditorium?

4. A shelf can hold 8 trophies. How many ways can the trophies be
arranged if there are 10 trophies?

5. You are considering 10 different colleges. Before you decide to apply


to the colleges, you want to visit some or all of them. In how many
orders can you visit 6 of the colleges?

6. You are considering 10 different colleges. Before you decide to apply


to the colleges, you want to visit some or all of them. In how many
orders can you visit all 10 colleges?

7. There are 8 horses in a race. If all we are concerned with the first,
second and third place finisher (the trifecta), how many different
orders
of finish are possible?

8. Suppose we are going to use the symbols (a, b, c, d, e, f, g, h) to form


a 5-character “password” having no repeated characters. How many
different passwords are possible?

9. In how many ways can 7 floats line up for the homecoming parade?

10. A corporation has 10 members on its board of directors. In how


many
different ways can it elect a president, vice-president, secretary and
treasurer?
11. For a segment of a radio show, a disc jockey can play 7 songs. If
there
are 13 songs to choose from, in how many ways can this segment be
arranged?

12. How many arrangements of 4 letters from the word COMBINE if no


letter is used more than once?

13. How many ways are there to place seven distinct pieces of art in a
row?

14. How many ways can a manager of a baseball team choose the lead-
off batter and second batter from a baseball team of nine players?

15. Maria has to visit 6 different places. In how many different ways can
she visit them?

45
Do you need more? Before you proceed to the next activity, kindly
check your score! If your score is:

11-15 You are doing great! You may proceed to the Lesson 3
after reviewing the missed items.
6-10 You are doing well but you need to proceed to the next
activities/exercises of this lesson.
1-5 You need to study harder. Ask help from your teacher or
parent or friend. Study the next examples and answer the
next activities.

Additional Activities
Answer the following problems below. Show your solutions.

1. If five alphabets are to be chosen from A, B, C, D, E, F and G such


that
repetition is not allowed then in how many ways it can be done?

2. How many 3 letter words can we make with the letters on the word
MATH?

3. In how many ways can 8 children arrange themselves in a row for a


picture taking?

4. A shelf can hold 9 books. How many ways can the trophies be
arranged
if there are 12 books?
5. How many different ways of management can there be to fill the
positions of principal and vice principal of a school knowing that there
are 11 eligible candidates?

6. Paul has 3 mobile phones in his desk. In how many different orders
can
the mobile phones be arranged?

7. If a class has 30 students, how many different arrangements can 5


students give a report presentation to the class?
8. Assume that you must select 3 people from a group of 20 people.
How
many ways are there to do this?

9. Given five letters (A, B, C, D, E). Find the following:


a. The number of four-letter word sequences.
b. The number of three-letter word sequences.
c. The number of two-letter word sequences.
10. How many permutations of the letters of the word ARTICLE have
consonants in the first and last positions?

46
Distinguishable and
Circular
Lesso
n Permutations
3
What I Need to Know
This lesson is created for you to solve problems in distinct or
distinguishable permutations and circular permutations. With the
problems given, you can either use reasoning to solve these types of
problems or you can use the distinguishable and the circular
permutations formula. Which the number of distinguishable
permutations, P, of n objects where p objects are alike, q objects are
n!
alike, r objects are alike, and so on, is P= while circular
p!q!r !…
permutations, P, of n objects is:P= ( n−1 ) ! .

You may SKIP this lesson or its activities if you already master
the skill and get a score of 11 out of 15 items in the pre-test.

What I Know
Find out how much you already know about this lesson. Choose
the letter that you think is correct and write your answer on the space
provided before each number on the separate sheet. Take note of the
items that you were not able to answer correctly and look for the right
answer as you go through this module.

___1. How many distinguishable permutations are possible with all the
letters of the word ELLIPSES?
A. 5 040 B. 5 060 C. 6 040 D. 6 060

___2. Find the number of different ways that a family of 6 can be


seated around a circular table with 6 chairs?
A. 90 B. 100 C. 120 D. 140
47
___3. Find the number of distinguishable permutations of the letters of
the word ALLAHABAD.
A. 7 540 B. 7 560 C. 8 540 D. 8 560

___4. In how many ways can 10 persons sit at a round table for a group
discussions?
A. 362 150 B. 362 160 C. 362 800 D. 362 880

___5. Determine the number of ways such that 5 men and 5 women be
seated at a round table if no two women are seated together.
A. 362 150 B. 362 160 C. 362 800 D. 362 880

___6. In how many distinguishable ways can the letters in


MASSACHUSETTS be written?
A. 64 864 800 B. 64 864 900
C. 65 000 000 D. 65 864 800

___7. How many ways can 8 people sit in a circle?


A. 5 000 B. 5 020 C. 5 040 D. 6 000

___8. In how many ways can 14 volleyball players be arranged in a


circular bunch?
A. 5 227 020 800 B. 5 227 020 900
C. 6 227 020 800 D. 6 227 020 900

___9. Find the number of different ways that a family of 7 can be


seated around a circular table with 7 chairs?
A. 420 B. 520 C. 620 D. 720

___10. How many distinguishable permutations are possible with all the
letters of the word OCCASION?
A. 10 080 B. 10 100 C. 10 110 D. 10 120

___11. Find the number of distinguishable permutations of the letters of


the word MISSISSIPPI?
A. 34 600 B. 34 650 C. 35 600 D. 35 650
___12. There are 12 people in a dinner gathering. In how many ways
can the host (one of the 12) arrange his guests around a dining
table if they can sit on any of the chairs?
A. 39 916 800 B. 39 916 900
C. 40 916 800 D. 40 916 900
48
___13. There are 12 people in a dinner gathering. In how many ways
can the host (one of the 12) arrange his guests around a dining
table if 3 people insist on sitting beside each other?
A. 1 177 260 B. 1 177 280
C. 2 177 260 D. 2 177 280
___14. A building contractor is planning to develop a subdivision that
consist of 6 one-story houses, 4 two-story houses, and 2 split-
level houses. In how many distinguishable ways can the houses
be arranged?
A. 11 860 B. 12 860 C. 13 860 D. 14 860

___15. Find the number of distinguishable permutations of the letters in


the word MALL.
A. 4 B. 8 C. 12 D. 16

If you all answered the problems above. Then, check your score!
If your score is:

11-15 You are doing great! You may proceed to the next
activity that can be found in What’s More after reviewing
the missed items.
6-10 You are doing well but you need to proceed to the next
activities/exercises of this lesson.
1-5 You need to study harder. Ask help from your teacher or
parent or friend. Study the next examples and answer the
different activities.

What’s In

Answer Me:

In how many distinguishable permutations does the word


PHILIPPINES have?
How many ways can you arrange 7 people seated in a circular
table?

Find out the answer to these questions while studying this lesson
and discover its application.
49
What’s New

Activity 18: Investigate Me!

1. How many distinguishable permutations exist for the letters in the


word
TENNESSEE?

Solution:
There are nine total letters in word, therefore, n=9
n1 =1T
n2 =4 E
n3 =2 N
n 4=2 S
Therefore, the number of distinguishable permutations equals:

Using the distinguishable permutations formula:

n!
P=
n1 ! n 2 ! n 3 ! n 4 !
9! 9 ×8 ×7 × 6 ×5 × 4 ×3 ×2 ×1 9 ×8 × 7 ×6 ×5 15 120
P= = = =
1 ! 4 ! 2 ! 2 ! 1 × 4 ×3 × 2× 1× 2×1 ×2 ×1 2× 2 4

¿ 3 780

Thus, there are 3 780 distinguishable permutations of the word


TENNESSEE.

2. Find the number of permutations of the letters of the word


STATISTICS.

Solution: Using Reasoning

There are 10 letters of the word. Assuming that the letters are
distinct, there are

P ( 10,10 )=10 ! permutations.

However, we have to take into consideration that the 3 S’s are


alike, the 3 T’s are alike, and the 2 I’s are also alike. The permutations
of the 3 S’s is P ( 3,3 )=3 !. The permutations of the 3 T’s is P ( 3,3 )=3 !.
The permutations of the 2 I’s is P ( 2,2 )=2 ! .

So, we must divide 10! by 3! 3! 2! In order to eliminate the


duplicates,
50
Thus,
10 !
P=
3 ! 3 ! 2!

10 ×9 × 8× 7 ×6 × 5× 4 × 3 ×2 ×1 10 × 9 ×8 ×7 × 5× 4 100 800
P= = = =50 400
3 × 2× 1×3 × 2× 1× 2× 1 2 2

This leads to the next rule.

The number of distinguishable permutations, P, of n objects


where p objects are alike, q objects are alike, r objects are alike, and so
n!
on, is: P= .
p!q!r !…

Let us now consider arrangement of objects in a circle, which we


call circular permutations.

3. a. In how many ways can 3 people be seated around a circular


table?

Solution: n=3

Notice that the arrangement 1-2-3 in (a) above is the same as 2-


3-1 and 3-1-2; meaning these 3 permutations are just one and the
same. Notice also that the arrangements 1-2-3, 3-2-1, and 2-1-3 are
also just the same as seen in (b).

So, the circular permutations, P, of 3 objects is:


6
P=
3

3!
P=
3
51
3 ×2 !
P=
3
P=2 !

We can also look at this way:

If there are 3 seats around the circular table, we can assign one
of the persons to a fixed seat and determine the number of ways that
the other two can be arranged. This way, we avoid counting again an
arrangement that resulted from a mere rotation. Thus, the number of
circular permutations of 3 objects is ( 3−1 ) !∨2 ! And the circular
permutations of n objects is ( n−1 ) ! .

b. In how many ways can 4 people be seated around a circular table?

Solution: ( n=4 ) Let us call the 4 people E, F, G, H

The arrangements are:

E-F-G-H E-F-H-G E-G-F-H E-G-H-B E-H-F-G E-H-G-F


F-G-H-E F-H-G-E G-F-H-E G-H-F-E H-F-G-E H-G-F-E
G-H-E-F H-G-E-F F-H-E-G H-F-E-G F-G-E-H G-F-E-H
H-E-F-G G-E-F-H H-E-G-F F-E-G-H G-E-H-F F-E-H-G

Observe that all the arrangements falling on the same column are
just the same because the 4 people are supposed to be seated around
a circular table. There are 24 arrangements in the list. Again, the
circular permutations, P, of 4 objects is:

24
P=
4

4!
P=
4

4 ×3 !
P=
4

P=3
52
Or simply,P= ( n−1 ) !

P= ( 4−1 ) !

P=3 !

P=6

The permutations of n objects arranged in a circle is: P= ( n−1 ) !

Activity 19: Can You Test Me?

Answer each permutations completely: Show your solution to the


separate sheet.

1. How many distinguishable permutations are possible with all the


letters of the word ELLIPSES?

2. A building contractor is planning to develop a subdivision that consist


of 6 one-story houses, 4 two-story houses, and 2 split-level houses.
In how many distinguishable ways can the houses be arranged?

3. Find the number of different ways that a family of 6 can be seated


around a circular table with 6 chairs?

4. How many distinguishable permutations exist for the letters in the


word CINCINNATI?

5. Determine the number of ways of selecting one or more letters from the
letters DDDDDD?

6. Find the number of distinguishable permutations of the letters in the


word MALL.

7. In how many ways can 5 people be arranged in a

8. Find the number of distinguishable permutations of the letters of the


word ALLAHABAD.

9. How many ways can 5 people sit in a circle?

10. In how many ways can 12 volleyball players be arranged in a


circular bunch?
53

What Is It
Distinguishable and Circular Permutations Defined

Distinguishable Permutations refers to the permutations of a set


of objects where some of them are alike. The number of distinguishable
permutations of n objects when p are alike, q are alike, r are alike, and
n!
so on, is given by P= .
p!q!r !…

For Example: Find the number of distinguishable permutations of the


letter OHIO.

Solution: Using Reasoning

There are 4 letters of the word OHIO, therefore, n=4 and p=2O .

Using the formula:


n!
P=
p!

4!
P=
2!

4 ×3 ×2 ×1
P=
2× 1

24
P=
2

P=12

Thus, there are 12 distinguishable permutations of the word OHIO.

What is Circular Permutations?

Circular Permutations refers to the different possible


arrangements of objects in a circle. The number of permutations P of n
objects around a circle is given by P= ( n−1 ) ! . If clockwise and anti-
clockwise circular permutations are considered to be the same, then it
( n−1 ) !
is P= .
2
Note: The number of circular permutations of n things when p are alike
and the rest different all taken at a time distinguishing clockwise and
( n−1 ) !
anti- clockwise arrangement is P= .
p!

54

For Examples:

1. 10 toppings of pizza are placed on the plate. How many ways can
they be arranged?

Solution:
P= ( n−1 ) !

P= (10−1 ) !

P=9!

P=9× 8 ×7 × 6× 5 ×4 ×3 ×2 ×1

P=362 880

Therefore, there are 362 880 possible arrangements of the 10


toppings of pizza.

2. In how many ways can 7 different charms be arranged on a circular


bracelet?

Solution: Using the ring permutation principle, there are:

( n−1 ) !
P=
2

( 7−1 ) !
P=
2

6!
P=
2

6 ×5 × 4 × 3× 2× 1
P=
2

720
P=
2

P=360
Thus, there are 360 possible arrangements of 7 different charms
in a circular bracelet.

55

3. Find the permutations of 6 people sitting around on a circular table if:

a. no restrictions

Solution:
P= ( n−1 ) !

P= (6−1 ) !

P=5 !

P=5 × 4 × 3× 2× 1

P=120

Therefore, there are 120 permutations if no restrictions of 6


people sitting around on a circular table.

b. 2 cannot sit next to each other

Solution:
P= ( n−1 ) !− ( n−2 ) ! ×2 !

P= (6−1 ) !−( 6−2 ) ! ×2 !

P=5 !−4 ! ×2 !

P=120−4 ×3 ×2 ×1 ×2 ×1

P=120−48

P=72

Thus, there are 72 permutations if 2 cannot sit next to each other.


56

What’s More
Activity 20: Guided/Controlled Practice

Answer each problems completely: Show your solution on the


sheets provided.

1. Ellie want to change her password which is ELLIE9 but with the
same
letters and number. In how many ways she can do that?

Solution:

Total number of letters = 6

Repeated letters = 2Ls and 2 Es

Number of times ELLIE9 can be rearranged by:

6! 6 × 5× 4 × 3 ×2 ×1 6 ×5 × 4 ×3 ×2 ×1
P= = =
2 ! 2! 2 ×1 ×2 ×1 4

¿ 6 ×5 ×3 × 2× 1=180.

But the password need to be changed. So the number of ways


new password can be made ¿ 180−1=179.

2. Find the number of distinguishable permutations of the letters in


MIAMI.

Solution:

MIAMI has 5 letters of which M and I are each repeated 2 times.


So, the number of distinguishable permutations is:
5! 5× 4 × 3 ×2 ×1 5× 4 × 3 ×2 ×1
P= = = =5 ×3 ×2 ×1
2 ! ×2 ! 2 ×1 ×2 ×1 4

¿ 30

3. In how many ways can 6 people be seated at a round table?

57
Solution:

The number of ways will be:

P= ( n−1 ) != ( 6−1 ) !=5!=5 × 4 ×3 × 2× 1=120

Therefore, there are 120 ways that 6 people can be seated at a


round table.

4. Find the number of ways in which 5 people A, B, C, D, and E can be


seated at a round table such that A and B must always sit together.

Solution:

If we wish to seat A and B together in all arrangements, we can


consider these two as one unit, along with 3 others. So effectively
we’ve to arrange 4 people in a circle, the number of ways being (4-1)!
or 3 ×2 ×1=6.

Thus, there are 6 arrangement such that A and B must always sit
together.

5. Find the number of different permutations of the letters of the word


MISSISSIPPI.

Solution:

The word MISSISSIPPI has 11 letters. If the letters were all


different there would have been 11! Different permutations. But
MISSISSIPPI has 4 S’s, 4 I’s, and 2 P’s that are alike. So, the answer
is:

11! 11×10 × 9× 8 ×7 × 6× 5 ×4 ×3 ×2 ×1
P= =
4 !4!2! 4 ×3 ×2 ×1 × 4 ×3 ×2 ×1 ×2 ×1

¿ 11 ×10 × 9× 7 ×5=34 650.

6. If a coin is tossed six times, how many different outcomes consisting


of
4 heads and 2 tails are there?

Solution:

Again, we have permutations with similar elements called


distinguishable permutations.

We are looking for permutations for the letters HHHHTT.

58
The answer is:
6! 6 × 5× 4 ×3 ×2 ×1 6 ×5 30
P= = = = =15.
4 ! 2! 4 × 3× 2× 1× 2× 1 2 2

7. In how many different ways can 4 nickels, 3 dimes, and 2 quarters be


arranged in a row?

Solution:

Assuming that all nickels are similar, all dimes are similar, and all
quarters are similar, we have permutations with similar elements.
Therefore, the answer is:

9! 9 × 8 ×7 ×6 × 5× 4 × 3 ×2 ×1 9× 8 ×7 ×5 2 520
P= = = =
4 ! 3 ! 2! 4 ×3 ×2 ×1 ×3 ×2 ×1 ×2 ×1 2 2

¿ 1 260

8. In how many different ways can five children hold hands to play “Ring
Around the Rosy”?

Solution:

The number of ways that five children hold hands to plat “Ring
Around the Rosy” is:

P= ( n−1 ) != (5−1 ) !=4 !=4 ×3 ×2 ×1=24.

9. In how many ways can three people be made to sit at a round table?

Solution:

P= ( n−r ) !=( 3−1 ) !=2!=2 ×1=2.

Thus, there are 2 ways that three people made to sit around the
table.
10. A stock broker wants to assign 20 new clients equally to 4 of its
salespeople. In how many different ways can this be done?

Solution:

This means that each sales person gets 5 clients. The problem
can be thought of as an ordered partitions problem. In that case, using
the formula we get,

59

20 !
P= =11732745 024.
5 ! 5 ! 5! 5 !

11. A shopping mall has a straight row of 5 flagpoles at its main entrance
plaza. It has 3 identical green flags and 2 identical yellow flags. How
many distinct arrangements of flags on the flagpoles are possible?

Solution:

The problem can be thought of as distinct permutations of the


letters GGGYY; that is arrangements of 5 letters, where 3 letters are
similar, and the remaining 2 letters are similar.

5! 5 × 4 × 3× 2× 1 5 × 4 20
P= = = = =10.
3 ! 2! 3 ×2 ×1 ×2 ×1 2 2

Just to provide a little more insight into the solution, we list all 10
distinct permutations:

GGGYY GYGGY YGGGY YYGGG


GGYGY GYGYG YGGYG
GGYYG GYYGG YGYGY

12. In how many ways can 4 people be seated around a circular table?

Solution:

The formula we are going to use is: P= ( n−1 ) !

So, the number of ways that 4 people be seated in circular table


is:

P= ( 4−1 ) !=3 !=3 ×2 ×1=6.

13. At a dinner party, 6 men and 6 women sit at a round table. In how
many ways can they sit if there are no restrictions?
Solution:

The number of different ways of 6 men and 6 women sit at a


round table with no restrictions is:

60

P= ( n−1 ) !

¿ ( 12−1 ) !

¿ 11 !

¿ 39 916 800

14. In how many ways can 8 differently coloured beads be threaded on a


string?

Solution:

As necklace can be turned over, clockwise and anti-clockwise


arrangements are the same.

( n−1 ) ! ( 8−1 ) ! 7! 7 × 6 ×5 × 4 ×3 × 2×1 5 040


P= = = = = =2 520.
2 2 2 2 2

Therefore, the arrangements of 8 differently coloured beads be


threaded on a string is 2 520.

15. At a dinner party, 6 men and 6 women sit at a round table. In how
many ways can they sit if men and women is alternate?

Solution:

The number of ways that a men and women sit at a round table
alternately is:

P= (6−1 ) ×6 !=5 ! ×6 !=120 ×720=86 400.

Since you already finish studying the activity above, let us check
your understanding by solving the different problems below.
Activity 21: Independent Practice

Answer the problems below. Show your solutions to the other


sheets.

1. ANNALIZA want to change her password which is ANNALIZA6 but


with the same letters and number. In how many ways she can do
that?

2. Find the number of distinguishable permutations of the letters in


BANANA.
61
3. In how many ways can 10 people be seated at a round table?

4. Find the number of ways in which 6 people L, M, N, O, P and Q can


be seated at a round table such that L and M must always sit
together.

5. Find the number of different permutations of the letters of the word


TALLAHASSEE.

6. If a coin is tossed seven times, how many different outcomes


consisting of 3 heads and 3 tails are there?

7. In how many different ways can 5 shirts, 4 shorts, and 3 shoes be


arranged in a row?

8. In how many different ways can six children ride a “Merry Go Around”
with six horses?

9. If a coin is tossed 10 times, how many different outcomes of 7 heads


and 3 tails are possible?

10. How many five-digit numbers can be made using two 6’s and three
7’s?

11. In how many ways can 5 men and 5 women be seated at a round
table
if there is no restrictions?

12. In how many ways can six trinkets be arranged on a chain?

13. There are 12 people in a dinner gathering. In how many ways can
the host (one of the 12) arrange his guests around a dining table if
they can sit on any of the chairs?

14. There are 12 people in a dinner gathering. In how many ways can
the host (one of the 12) arrange his guests around a dining table if 3
people insist on sitting beside each other?

15. There are 12 people in a dinner gathering. In how many ways can
the host (one of the 12) arrange his guests around a dining table if 2
people refuse to sit beside each other?

62
I hope you answered all of the item! To check your score!
If your score is:

11-15 You are doing great! You may proceed to the next
activity, the Assessment (Activity 8) after reviewing the
missed items.
6-10 You are doing well but you need to proceed to the next
activities/exercises of this lesson.
1-5 You need to study harder. Ask help from your teacher or
parent or friend. Study the next examples and answer the
next activities.

What I Have Learned


Activity 22: Do I Really Learned?

Answer the following problems solving. Show your solution to the


separate sheet.

1. In how many distinguishable ways can the letters in


MASSACHUSETTS be written?

2. From the word MATHEMATICS, in how many distinguishable


permutations can be formed?

3. In how many ways can 10 persons sit at a round table for a group
discussions?

4. How many distinguishable permutations exist for the letters in the


word COMMITTEE?

5. Determine the number of ways such that 5 men and 5 women be


seated at a round table if no two women are seated together.

63

What I Can Do

Activity 23: Finish Me!

Solve the following problems. Show your solution to the separate


sheet.

1. How many distinguishable permutations are possible with all the


letters of the word OCCASION?

2. In a playground, 3 sisters and 8 other girls are playing together. In a


particular game, how many ways can all the girls be seated in a
circular order so that the three sisters are not seated together?

3. Find the number of different ways that a family of 7 can be seated


around a circular table with 7 chairs?

4. How many distinguishable permutations exist for the letters in the


word PHILIPPINES?

5. Determine the number of ways of selecting one or more letters from the
letters ZZZZZZZ?

6. Determine the number of ways such that 5 men and 5 women be seated
at a round table if no two women are seated together.
7. Find the number of ways in which 6 people L, M, N, O, P, A can be
seated at a round table, such that L and M must always sit together.

8. Find the number of permutations of the letters of the word


PERMUTATIONS.

9. How many ways can 8 people sit in a circle?

10. In how many ways can 14 volleyball players be arranged in a


circular bunch?

64

Assessment
Activity 24: Guided Assessment

Answer each problems completely: Show your solution on the


sheets provided.

1. If a coin is tossed six times, how many different outcomes consisting


of
4 heads and 2 tails are there?

Solution:

Again, we have permutations with similar elements called


distinguishable permutations.

We are looking for permutations for the letters HHHHTT.

The answer is:


6! 6 × 5× 4 ×3 ×2 ×1 6 ×5 30
P= = = = =15.
4 ! 2! 4 × 3× 2× 1× 2× 1 2 2

2. A stock broker wants to assign 20 new clients equally to 4 of its


salespeople. In how many different ways can this be done?

Solution:

This means that each sales person gets 5 clients. The problem
can be thought of as an ordered partitions problem. In that case, using
the formula we get,
20 !
P= =11732745 024.
5 ! 5 ! 5! 5 !

3. In how many different ways can 4 nickels, 3 dimes, and 2 quarters be


arranged in a row?

Solution:

Assuming that all nickels are similar, all dimes are similar, and all
quarters are similar, we have permutations with similar elements.
Therefore, the answer is:

65

9! 9 × 8 ×7 ×6 × 5× 4 × 3 ×2 ×1 9× 8 ×7 ×5 2 520
P= = = =
4 ! 3 ! 2! 4 ×3 ×2 ×1 ×3 ×2 ×1 ×2 ×1 2 2

¿ 1 260

4. In how many ways can three people be made to sit at a round table?

Solution:

P= ( n−r ) !=( 3−1 ) !=2!=2 ×1=2.

Thus, there are 2 ways that three people made to sit around the
table.

5. Ellie want to change her password which is ELLIE9 but with the
same
letters and number. In how many ways she can do that?

Solution:

Total number of letters = 6

Repeated letters = 2Ls and 2 Es

Number of times ELLIE9 can be rearranged by:

6! 6 × 5× 4 × 3 ×2 ×1 6 ×5 × 4 ×3 ×2 ×1
P= = =
2 ! 2! 2 ×1 ×2 ×1 4

¿ 6 ×5 ×3 × 2× 1=180.
But the password need to be changed. So the number of ways
new password can be made ¿ 180−1=179.

6. At a dinner party, 6 men and 6 women sit at a round table. In how


many
ways can they sit if men and women is alternate?

Solution:

The number of ways that a men and women sit at a round table
alternately is:

P= (6−1 ) ×6 !=5 ! ×6 !=120 ×720=86 400.

7. Find the number of different permutations of the letters of the word


MISSISSIPPI.

66
Solution:

The word MISSISSIPPI has 11 letters. If the letters were all


different there would have been 11! Different permutations. But
MISSISSIPPI has 4 S’s, 4 I’s, and 2 P’s that are alike. So, the answer
is:

11! 11×10 × 9× 8 ×7 × 6× 5 ×4 ×3 ×2 ×1
P= =
4 !4!2! 4 ×3 ×2 ×1 × 4 ×3 ×2 ×1 ×2 ×1

¿ 11 ×10 × 9× 7 ×5=34 650.

8. In how many ways can 8 differently coloured beads be threaded on a


string?

Solution:

As necklace can be turned over, clockwise and anti-clockwise


arrangements are the same.

( n−1 ) ! ( 8−1 ) ! 7! 7 × 6 ×5 × 4 ×3 × 2×1 5 040


P= = = = = =2 520.
2 2 2 2 2

Therefore, the arrangements of 8 differently coloured beads be


threaded on a string is 2 520.

9. Find the number of ways in which 5 people A, B, C, D, and E can be


seated at a round table such that A and B must always sit together.
Solution:

If we wish to seat A and B together in all arrangements, we can


consider these two as one unit, along with 3 others. So effectively
we’ve to arrange 4 people in a circle, the number of ways being (4-1)!
or 3 ×2 ×1=6.

Thus, there are 6 arrangement such that A and B must always sit
together.

10. At a dinner party, 6 men and 6 women sit at a round table. In how
many ways can they sit if there are no restrictions?

Solution:

The number of different ways of 6 men and 6 women sit at a


round table with no restrictions is:

67

P= ( n−1 ) !

¿ ( 12−1 ) !

¿ 11 !

¿ 39 916 800

11. In how many ways can 6 people be seated at a round table?

Solution:

The number of ways will be:

P= ( n−1 ) != ( 6−1 ) !=5!=5 × 4 ×3 × 2× 1=120

Therefore, there are 120 ways that 6 people can be seated at a


round table.

12. In how many ways can 4 people be seated around a circular table?

Solution:

The formula we are going to use is: P= ( n−1 ) !

So, the number of ways that 4 people be seated in circular table


is:
P= ( 4−1 ) !=3 !=3 ×2 ×1=6.

13. Find the number of distinguishable permutations of the letters in


MIAMI.

Solution:

MIAMI has 5 letters of which M and I are each repeated 2 times.


So, the number of distinguishable permutations is:

5! 5× 4 × 3 ×2 ×1 5× 4 × 3 ×2 ×1
P= = = =5 ×3 ×2 ×1
2 ! ×2 ! 2 ×1 ×2 ×1 4

¿ 30

14. A shopping mall has a straight row of 5 flagpoles at its main entrance
plaza. It has 3 identical green flags and 2 identical yellow flags. How
many distinct arrangements of flags on the flagpoles are possible?

68
Solution:

The problem can be thought of as distinct permutations of the


letters GGGYY; that is arrangements of 5 letters, where 3 letters are
similar, and the remaining 2 letters are similar.

5! 5 × 4 × 3× 2× 1 5 × 4 20
P= = = = =10.
3 ! 2! 3 ×2 ×1 ×2 ×1 2 2

Just to provide a little more insight into the solution, we list all 10
distinct permutations:

GGGYY GYGGY YGGGY YYGGG


GGYGY GYGYG YGGYG
GGYYG GYYGG YGYGY

15. In how many different ways can five children hold hands to play
“Ring
Around the Rosy”?

Solution:

The number of ways that five children hold hands to plat “Ring
Around the Rosy” is:

P= ( n−1 ) != (5−1 ) !=4 !=4 ×3 ×2 ×1=24.


Good job! You are done studying with activity 17. And now, kindly
answer the activity below to inspect if you comprehend what you are
reading.

Activity 25: Independent Assessment

Solve the following problems below. Show your solutions to the


other sheets.

1. How many ways can eight people sit around a round table?

2. Calculate the circular permutations of 7 elements.

3. A couple wants to plant some shrubs around a circular walkway. They


have seven different shrubs. How many different ways can the shrubs
be planted?

69
4. Find the number of ways in which 5 people (Ian, Mark, Carlo, Lowie,
and Joel) can be seated at a round table, such that Carlo and Lowie
must not sit together.

5. In how many ways can 3 men and 3 ladies be seated at a round table
such that no two men are seated together?

6. Find the number of different permutations of the letters of the word


ASSESSMENT.

7. Find the distinguishable permutations of the letters in the word


ACCESSORIES.

8. In how many ways can five keys be put on the key ring?

9. How many six-digit numbers can be made using two 2’s and two 3’s?

10. If a coin is tossed 5 times, how many different outcomes of 3 heads


and 2 tails are possible?

11. How many different ways can 3 pennies, 2 nickels and 5 dimes be
arranged in a row?

12. In how many ways can 11 people be made to sit at a round table?

13. Find the number of different permutations of the letters of the word
BILLIONAIRE.

14. Find the distinguishable permutations of the letters in the word


BALLISTICS.

15. Find the distinguishable permutations of the letters in the word


KANAKANAK.

Do you need more activity? But before you proceed to the next
activity, kindly check your score! If your score is:

11-15 You are doing great! You may proceed to the Post-test
after reviewing the missed items.
6-10 You are doing well but you need to proceed to the next
activities/exercises of this lesson.
1-5 You need to study harder. Ask help from your teacher or
parent or friend. Study the next examples and answer the
next activities.

70

Additional Activities
Answer the following problems below. Show your solutions.

1. Jessie wants to plant 10 plants along the sidewalk in her front yard.
She has 3 rose bushes, 4 daffodils, and 3 lilies. In how many
distinguishable ways can the plants be arranged?

2. In how many ways can 4 red, 3 yellow and 2 green discs be


arranged
in a row if the discs of the same colour are indistinguishable?

3. The signal mast of a ship can raise nine flags at one time (3 red, 2
blue, and 3 green). How many different signals can be
communicated
by the placement of these nine flags?

4. How many nine-digit numbers can be formed with the numbers 2, 2,


2, 3, 3, 3, 3, 4, 4?

5. In how many ways can 5 people be arranged in a circle?


6. How many different ways can 3 red, 2 yellow and 2 blue bulbs be
arranged in a string of Christmas tree lights with 9 sockets?

7. In how many ways can the six letters of the word “MAMMAL” be
arranged in a row?

8. How many different ways can 7 different items be arranged on a


circular tray?

9. Find the number of different permutations of the letters of the word


ADDITIONAL.

10. Find the distinguishable permutations of the letters in the word


CORPORATION.

71
SUMMARY/SYNTHESIS/GENERALIZATION
This module was about solving problems involving permutations
and its application in real-life situations. The every lesson under this
module was provided you with opportunities to solve differentiated
problems using the fundamental counting principle, the permutations
formula and the distinguishable and circular permutations formula. In
addition, you were given a chance to know how to solve problems
involving permutations that apply the skills and knowledge to formulate
conclusions and making decisions. Your understanding of this lesson
as long as the concepts of Mathematics that you already learned will
help you understand the next topic, combinations.
72
Assessment

Post-Test

Find out how much you already know about this module. Choose
the letter that you think is correct and write your answer on the space
provided before the number. . Take note of the items that you were
not able to answer correctly and look for the right answer as you go
through this module.

___1. In a town fiesta dancing competition with 12 contestants, in how


many ways can the organizer arrange the first three dancers?
A. 132 B. 990 C. 1320 D. 1716

___2. Find the number of distinguishable permutations of the letters of


the word EDUCATED.
A. 1680 B. 10 080 C. 20 160 D. 40 320

___3. How many different 3-digit numbers can be formed from the
digits 1,2,3,5,6,7 if repetition of digits is not allowed?
A. 120 B. 360 C.720 D.840

___4. Find the number of distinguishable permutations of the letters of


the word PASS.
A. 4 B. 12 C. 36 D. 144

___5. Which of the following expressions represents the number of


distinguishable permutations of the letters of the word
HAPPINESS?
9! 9! 9!
A. 9! B. C. D.
2! 5! 2 ! 2! 2 !

___6. Six athletes will participate in a gymnastics competition. In how


many ways can they win first, second, and third places, if there
are no ties?
A. 110 B. 120 C. 210 D. 150

73
___7. Ms. Dela Cruz wants to produce different sets of test questions
for her essay test. If she plans to do this by putting together 3 out
of 5 questions she prepared, how many different sets of
questions could she construct?
A. 10 B. 20 C. 60 D. 80

___8. In the recently conducted SEA Games, there are 11 contenders


in a swimming events. How many ways can the gold, the silver,
and the bronze medals be won?
A. 856 B. 878 C. 950 D. 990

___9. In how many ways can 8 people be seated around a circular


table if two of them insist on sitting beside each other?
A. 360 B. 720 C. 1440 D. 5040

___10. Lydia, together with her five friends want to arrange themselves
around a round table. How many ways can they do it?
A. 720 B. 120 C. 60 D. 30

___11. Determine the different ways can 6 potted plants be arranged in


a row?
A. 210 B. 540 C. 550 D. 720

___12. How many different 4-digit even numbers can be formed from
the digits 1, 3, 5, 6, 8, and 9 if no repetition of digits is allowed?
A. 1 680 B. 840 C. 420 D. 120

___13. In how many different ways can 5 different-colored horses be


positioned in a carousel?
A. 24 B. 120 C. 720 D. 800

___14. In a room, there are 10 chairs in a row. In how many ways can
5
students be seated in consecutive chairs?
A. 120 B. 720 C. 25 600 D. 30 240

___15. In how many ways can 4 people be seated around a circular


table?
A. 2 B. 6 C. 12 D. 24
74

Answer Keys

Pre-Assessment
1. B 6. D 11. B
2. C 7. A 12. A
3. D 8. D 13. D
4. A 9. B 14. B
5. C 10. B 15. B

Lesson 1: What I Know


1. A 6. A 11. A
2. C 7. B 12. A
3. C 8. D 13. C
4. B 9. C 14. B
5. D 10. D 15. A

Activity 2: Find Me Out!


1. 60 6. 24
2. 720 7. 2 520
3. 720 8. 6 227 020 800
4. 5 040 9. 151 200
5. 6 10. 720

Activity 4: Independent Practice


1. 64 6. 56 11. 75
2. 120 7. 6 12. 56
3. 24 8. 15 13. 1
024
4. 720 9. 30 14. 720
5. 12 10. 125 15. 80

Activity 5: Accomplish Me!


1. 280 3. 360 360 5. 42 840
2. 362 880 4. 360
75

Activity 6: I Can Do It!


1. 24 6. 32
2. 42 7. 120
3. 180 8. 240
4. 6 9. 720
5. 1 320 10. 2 880

Activity 8: Independent Assessment


1. 80 6. 15 11. 8 315
2. 20 7. 576 12. 37
856
3. 9 8. 40 320 13. 1 024
4. 60 9. 81 000 000 14. 59
049
5. 12 10. 17 576 000 15. 1 944

Activity 9: Additional Activities


1. 6 760 000 3. 120 5. 6
2. 3 276 000 4. 56

Lesson 2: What I know


1. C 6. C 11. A
2. D 7. B 12. D
3. A 8. D 13. C
4. D 9. C 14. D
5. A 10. B 15. A

Activity 11: Let’s Try This


1. 24 6. 32
2. 42 7. 120
3. 180 8. 240
4. 6 9. 720
5. 1 320 10. 2 880

Activity 13: Independent Practice


1. 210 6. 151 200 11. 60
2. 95 040 7. 1 367 520 12. 24
3. 358 800 8. 504 13. 840
4. 90 9. 360 14. 720
5. 6 375 600 10. 11 880 15. 6 840
76
Activity 14: Warm That Brain!
1. 3 360 3. 151 200 5. 42 840
2. 362 880 4. 6 720

Activity 15: Tell Me If I Can Do It?


1. 56 6. 210
2. 604 800 7. 168
3. 120 8. 480
4. 144 9. 60
5. 24 10. 576

Activity 17: Independent Assessment


1. 362 880 6. 3 628 800 11. 8 648 640
2. 32 736 7. 336 12. 840
3. 40 320 8. 2 520 13. 5 040
4. 90 9. 5 040 14. 72
5. 151 200 10. 5 040 15. 720

Lesson 2: Additional Activities


1. 2 520 6. 6
2. 24 7. 17 100 720
3. 40 320 8. 6 840
4. 79 833 600 9. a. 12 b. 60 c.
20
5. 10 10. 1 440

Lesson 3: What I know


1. A 6. A 11. B
2. C 7. C 12. A
3. B 8. C 13. D
4. B 9. D 14. C
5. D 10. A 15. C

Activity 19: Can You Test Me?


1. 5 040 6. 12
2. 13 860 7. 12
3. 120 8. 7 560
4. 50 400 9. 24
5. 6 10. 39 916 800
77

Activity 21: Independent Practice


1. 15 120 6. 140 11. 362 880
2. 30 7. 27 720 12. 10
3. 362 880 8. 120 13. 39 916
800
4. 6 9. 120 14. 2 177 280
5. 831 600 10. 10 15. 32 659 200

Activity 22: Do I Really Learned?


1. 64 864 800 3. 362 880 5. 362
160
2. 4 989 600 4. 45 360

Activity 23: Finish Me!


1. 10 080 6. 362 160
2. 3 386 880 7. 48
3. 720 8. 239 500 800
4. 1 108 800 9. 5 040
5. 7 10. 6 227 020 800

Activity 25: Independent Assessment


1. 5 040 6. 75 600 11. 2 520
2. 720 7. 1 663 200 12. 3 628 800
3. 720 8. 60 13. 3 326 400
4. 6 9. 6 14. 453 600
5. 120 10. 10 15. 5 040

Lesson 3: Additional Activities


1. 1 050 6. 15 120
2. 1 260 7. 60
3. 5 040 8. 720
4. 1 260 9. 907 200
5. 24 10. 3 326 400
78
References

 https://sidebysideconsulting.com/2017/11/16/5-counting-principles-
every-preschooler-needs-to-know/
 https://www.superprof.co.uk/resources/academic/maths/probability/com
binatorics/permutations-word-problems.html
 https://www.priklady.eu/en/mathematics/combinatorics/permutations.al
ej
 https://math.info/Algebra/Distinguishable_Permutations/
 https://www.sanfoundry.com/discrete-mathematics-questions-answers-
circular-permutations/
 https://www.google.com/search?
q=solving+problems+in+circular+permutations+with+solutions&hl=en-
US&source=lnms&tbm=isch&sa=X&ved=2ahUKEwi--
PLf9ebpAhUNPnAKHUN4DAIQ_AUoAnoECA0QBA&biw=1366&bih=6
13#imgrc=dJjrNL5OUVlxuM
 https://www.onlinemathlearning.com/permutations-math.html
 https://www.onlinemath4all.com/fundamental counting principle-
worksheets-with-answers.html
 https://www.statisticshowto.com/Fundamental-Counting-Principle
 https://www.thefreedictionary.com
 https://www.classzone.com
 Department of Education Mathematics 10 Learner’s Guide
 Department of education Mathematics 10 Teacher’s Guide
 https://www. Coursehero.com/file/53695187/Permutations-Practice-
Worksheet-1docx/
 https://www.analyzemath.com
 https://www.math.fsu.edu/wooland/hm2ed/Part1Module4/Part1Module4
.pdf
 https://www.probabilityformula.org
 https://intmath.org
 https://www.mathworksheetsland.com/22 permset.html
 https//www.enetlearning.org
 https://www.khanacademy.org
 https://study.com.academy/lesson/permutations-combinations-
problems-practice.html
 https://mi01000971.schoolwises.net
79

For inquiries and feedback, please write or call:

Department of Education –Learning Resources Management and


Development Center (LRMDC)

DepEd Division of Bukidnon


Sumpong, Malaybalay City, Bukidnon
Telefax: ((08822)855-0048
E-mail Address: bukidnon@deped.gov.ph

You might also like